Praxis - Elementary Education: Multiple Subjects (5001)
Capitalization
Conventions for when a word should start with a capital (uppercase) letter, capitalize the following words and phrases: proper nouns and proper adjectives
We can convert measurements from one unit to another using...
Conversion factors
Unit reasoning helps us make sense of measurements by...
Converting between measurement units and making reasonable estimates
medicinal plants
these were used by Enslaved Africans in their native land, they adapted to their new surroundings by learning the use of New World plants through experimentation and from the indigenous inhabitants
Comparative form of an adjective
"-er," used to compare two things
Superlative form of an adjective
"-est," used to compare many things
Area
"A" of a polygon is the amount of flat space inside the polygon's edges, it is measured in square units such as square miles; different polygons have their own area formulas
Perimeter
"P" of a polygon is the total length around the polygon's edges, when we add up all the side lengths of a polygon, we get its perimeter. For the figure below, the perimeter is:
Volume
"V" of an object is the amount of space that object occupies, it is measured in cubic units such as cubic centimeters
How do we calculate percent changes?
% change = (final - initial / initial) x 100
At a craft fair, Tony bought 3 items, Isabella bought 1 item, and Tanisha bought 5 times as many items as Tony and Isabella did combined. Which of the following represents the number of items Tanisha bought?
(3 + 1) x 5
A student plans to simultaneously toss a fair number cube, with faces numbered 1 through 6, and a fair coin. What is the probability that the cube will land with the face numbered 4 up and the coin will land heads up? (A) 1/12 (B) 1/8 (C) 1/6 (D) 2/3
(A) 1/12 The correct answer is (A). The question requires an understanding of how to interpret probabilities relative to likelihood of occurrence. The coin has 2 possible outcomes: heads or tails. The probability of the coin landing heads up is 1 out of 2, or 1/2. The cube has 6 possible outcomes: 1, 2, 3, 4, 5, or 6. The probability of the face numbered 4 landing up is 1 out of 6, or 1/6. To find the combined probability, multiply the two independent probabilities together: 1/2 x 1/6 = 1/12.
The only prime factors of a certain number are 2, 3, and 7. Which of the following could be the number? (A) 18 x 28 (B) 20 x 21 (C) 22 x 63 (D) 24 x 35
(A) 18 x 28 The correct answer is (A). The question requires an understanding of how to find factors and multiples of numbers. The prime factorization of 18 is 2 x 3² and the prime factorization of 28 is 2² x 7. So the prime factorization of 18 x 28 is 2^3 × 3^2 × 7^1.
Which of the following technology-based tools best facilitates both personal writing and written discussion about the writing? (A) Blogging programs (B) Interactive gaming (C) Slide-share programs (D) Interactive whiteboards
(A) Blogging programs The correct answer is (A). Blogging programs facilitate personal writing and typically have open forums that encourage readers to respond to the writing with written discussion. Interactive gaming (B) encourages interaction and perhaps oral discussion but does not promote writing. Slide-sharing (C) may result in written commentary, but the response is to an image, not a written piece. Interactive whiteboards (D) are a tool to help students visualize and participate in certain aspects of writing, but they do not facilitate personal writing.
An English-language learner who is capable of matching pictures with words and phrases from a story but cannot yet use those pictures to recreate the sequence of a story is functioning at which of the following language proficiency levels? (A) Entering (B) Developing (C) Expanding (D) Bridging
(A) Entering The correct answer is (A). English-language learners at the (A) "entering" stage of language proficiency can process and produce the English needed to successfully engage in the reading activities mentioned but cannot yet identify main ideas or sequence pictures from oral stories. These skills come later, appearing first while the student is in the (B) "developing" level. Once they have reached the (C) "expanding" and (D) "bridging" proficiency levels, English-language learners possess even more advanced skills, such as finding details that support main ideas.
Which TWO of the following syllable types are present in the word "remarkable"? (A) Open (B) Closed (C) Vowel team (D) R-controlled
(A) Open (D) R-controlled The correct answers are (A) and (D). (A) is correct because the syllable "re" is an open syllable. An open syllable ends in a long vowel sound produced by a single vowel. (D) is correct because the syllable "mar" is an r-controlled syllable. The "r" controls the vowel sound, causing the "a" to have a unique sound. Neither (B) nor (C) is present in the word "remarkable"; in (B), a closed syllable ends in a consonant, and in (C), a vowel team consists of two or more vowels together that make a unique sound.
Which of the following is the most appropriate strategy for using easy books to increase fluency in a nonfluent student? (A) Providing opportunities for the nonfluent student to read self-selected easy books to a younger student (B) Asking a younger, more fluent reader to read an easy book aloud to the nonfluent student (C) Assigning an easy-to-read nonfiction book to the nonfluent student for independent reading (D) Encouraging other students to interrupt and correct when the nonfluent student is reading easy books aloud
(A) Providing opportunities for the nonfluent student to read self-selected easy books to a younger student The correct answer is (A). Fluency refers to reading smoothly, quickly, and with expression. (A) offers the nonfluent student opportunities to engage in meaningful literary experiences while gaining courage and self-esteem, and while also experiencing ownership. (B) and (D) are incorrect because a nonfluent student might feel intimidated and discouraged by either hearing a younger, more fluent reader or by having his/her classmates interrupt and correct him/her. While assigning an easy-to-read book (C) may help give the student practice, it takes away from the student's autonomy and may detract from the enjoyment.
Leave me, O love which reaches but to dust; And thou, my mind, aspire to higher things; Grow rich in that which never taketh rust, Whatever fades but fading pleasure brings. The lines above comment on the speaker's desire to... (A) seek out immediate pleasures (B) enrich himself (C) reject that which is transitory (D) revive the past
(C) reject that which is transitory The correct answer is (C). The word "transitory" refers to change, and the speaker mentions a desire to reject things that turn to dust, acquire dust, and start to fade. These are all types of change. (A) and (B) are incorrect because although the speaker may seek pleasure and although that pleasure may enrich him/ her, he/she is more interested in things that "never taketh rust," or change. (D) is incorrect because these lines suggest the speaker is looking toward the future, rather than dwelling in the past.
Mike loves playing football. He is the quarterback for his team, and he knows when to call the right plays. He hates having to miss a practice, and his teammates are frustrated when he's not there. They always say, "Where is Mike? We are lost without him." The team usually has a good practice even if he is not there, but they miss out on practicing key plays. Which of the following best explains how the reader knows that the passage is from a thirdperson narrative? (A) The narrator is only an observer of the action in the passage. (B) The narrator is participating in the dialogue in the passage. (C) The narrator discloses only his or her thoughts and feelings in the passage. (D) The narrator uses the present tense to discuss the conflict in the passage.
(A) The narrator is only an observer of the action in the passage. The correct answer is (A). The narrator of the passage is an observer, not a participant, in the action and dialogue of the passage. This is characteristic of a thirdperson narrative. (B) and (C) are incorrect because the narrator neither participates in the dialogue nor discloses his/her thoughts and feelings. In (D), while the narrator uses present tense, this is not an indicator of point of view.
5 < 7 - p Which of the following is equivalent to the inequality above? (A) p < 2 (B) p < -2
(A) p < 2 The correct answer is (A). The question requires an understanding of how to solve multistep one-variable linear inequalities. Adding −5 to both sides of the inequality yields the equivalent inequality 0 < - p. Adding "p" to both sides of the new inequality yields the equivalent inequality p < 2.
In a student discussion about whether the school cafeteria should stop selling junk food, which of the following statements best demonstrates active listening? (A) "In my opinion, it would be a mistake to remove junk food from the cafeteria because no one would eat there anymore." (B) "Raul thinks that our health should come before eating what we love, but Lacey argues that the schools should not take away our right to choose." (C) "How many of you would actually buy lunch if the cafeteria stopped selling junk food?" (D) "What if we write a formal complaint to the superintendent to voice our opinion on the food in the cafeteria?"
(B) "Raul thinks that our health should come before eating what we love, but Lacey argues that the schools should not take away our right to choose." The correct answer is (B). The statement demonstrates paraphrasing of others' statements. An active listener spends more time listening than talking, which paraphrasing demonstrates. Statement (A) neither acknowledges previous opinions nor asks for any further ones. While statement (C) asks for opinions, it does not indicate that the speaker will actively listen to the replies. Though statement (D) is worded as a question, it is a statement of an idea that may or may not have been developed through active listening.
Riding on a school bus are 20 students in ninth grade, 10 students in tenth grade, 9 students in eleventh grade, and 7 students in twelfth grade. Approximately what percent of the students on the bus are in ninth grade? (A) 23% (B) 43% (C) 46% (D) 76%
(B) 43% The correct answer is (B). The question requires an understanding of percent as a rate per 100. Percent refers to how many out of one hundred or, in decimal form, how many hundredths. To find a percent, divide the group (20) by the total (46) and round the decimal to the hundredths place (0.43). This is 43 hundredths or 43/100 or 43%.
What is the area, in square units, of the figure above? (A) 32 (B) 52 (C) 64 (D) 104
(B) 5 The correct answer is (B). The question requires an understanding of how to find the area of polygons, including those with fractional side lengths. The figure is composed of a rectangle and a triangle. The rectangle has length 10 and width 4, so its area is 40. The triangle can be thought of as having a base of 4 and a height of 6. Its area is 1/2(4)(6), or 12. The combined area is, therefore, 40 + 12, or 52.
Which of the following is most commonly used in digital text to give a reader access to additional information about a topic? (A) A citation (B) A hyperlink (C) An index (D) A glossary
(B) A hyperlink The correct answer is (B). The reader can follow the link provided to easily seek more information. In (A), a citation, the reader would have to spend time looking up the correct source. (C) is incorrect because the index will only provide the reader information about the text itself and not provide additional information. (D), a glossary, will provide definitions for unknown words but is limited in its function.
Which of the following is most typically included in the conclusion of an oral presentation? (A) An expansion of the thesis (B) A summarization of the main points (C) An attempt to build rapport with the audience (D) A move to gain the audience's attention
(B) A summarization of the main points The correct answer is (B). The conclusion of an oral presentation usually contains a clear summary of the main points to reinforce the presentation's goal. An expansion of the thesis (A) should occur during the body of the presentation, and both (C) and (D) should occur at the beginning of the presentation.
Which TWO of the following can be classified as expository writing? (A) A short story (B) A technical speech (C) A personal diary (D) A scientific report (E) An editorial commentary
(B) A technical speech (D) A scientific report The correct answers are (B) and (D). A technical speech and a research report both require that information be collected and synthesized. A short story (A) involves narrative writing, while a personal diary (C) and an editorial commentary (E) involve the writer's thoughts and opinions but require no facts or information.
A student whose writing shows an awareness of spacing (though spaces between words are not even), makes sporadic use of proper capitalization, and contains some invented spelling is most likely functioning at which of the following stages of developmental writing? (A) Emergent (B) Transitional (C) Conventional (D) Proficient
(B) Transitional The correct answer is (B). Students at the transitional level are beginning to have a more formal sense of print conventions, letters, words, and sentences. The characteristics in the list describe a student performing at the transitional level. Students at the (A) emergent level may use letters or letter-like symbols but are not yet familiar with conventions of print or spelling. At the (C) conventional level, students are able to select types of writing to suit their purpose and have more control of structure, punctuation, and spelling, and (D) proficient writers have developed a personal style and a large vocabulary, and their writing is also cohesive and coherent.
1. The teacher from Nebraska displayed Native American artifacts to her class. 2. The teacher displayed Native American artifacts from Nebraska to her class. The meaning of sentence 1 differs from that of sentence 2 in that the... (A) sentences do not have the same simple predicate (B) adjective phrase "from Nebraska" modifies different nouns (C) subject of sentence 1 is "teacher," while the subject of sentence 2 is "artifacts" (D) first sentence ends in a prepositional phrase, while the second sentence does not
(B) adjective phrase "from Nebraska" modifies different nouns The correct answer is (B). The meaning of the two sentences differs because in sentence 1 the teacher is from Nebraska, while in sentence 2 the Native American artifacts are from Nebraska. Thus, the placement of the adjective phrase "from Nebraska" after two different nouns changes the meaning of the sentences. (A) is incorrect because the simple predicate, "displayed," remains the same in both sentences. (C) is incorrect because the teacher is the subject of both sentences, and in (D), both sentences do end in the same prepositional phrase.
Two hamsters sat in the cage side by side; a furtive, timid one and a glossy, bold one watched each other warily. The sentence above is an example of a... (A) simple sentence (B) compound sentence (C) complex sentence (D) compound-complex sentence
(B) compound sentence The correct answer is (B). This sentence has two independent clauses joined by a semicolon, including one independent clause with a compound subject. Without dependent clauses, the sentence cannot be characterized as either (C) complex or (D) compound-complex. A simple sentence (A) contains only one independent clause.
Leave me, O love which reaches but to dust; And thou, my mind, aspire to higher things; Grow rich in that which never taketh rust, Whatever fades but fading pleasure brings. In line 1 "dust" serves as a metaphor for (A) ignorance (B) death (C) loneliness (D) confusion
(B) death The correct answer is (B). A metaphor is a type of figurative language in which one image or idea is connected with another. In literature, the word "dust" is often associated with death because life-forms decay into soil after death. The speaker indicates that love is something that will "taketh rust," and therefore, it is a comparison about age, not of (A) ignorance or (D) confusion. The speaker is not concerned about (C) loneliness, but rather about finding pleasure in what will remain constant.
A fourth-grade class started working on math worksheets at 1:30 P.M. and stopped working at 3:10 P.M. How long did the class work on the math worksheets? (A) 40 minutes (B) 80 minutes (C) 100 minutes (D) 120 minutes
(C) 100 minutes The correct answer is (C). The question requires an understanding of how to solve problems involving the measurement of elapsed time. Between 1:30 P.M. and 3:10 P.M. there are 1 hour and 40 minutes, or 100 minutes.
In the expression 4x² + 7, what is the degree of 4x²? (A) 0 (B) 1 (C) 2 (D) 4
(C) 2 The correct answer is (C). The question requires an understanding of how to use mathematical terms to identify parts of expressions and describe expressions. The degree of a monomial is the sum of the exponents of the variables that appear in it. In 4x² there is only one variable, x, and its exponent is 2.
How many phonemes are in the word "ball"? (A) 1 (B) 2 (C) 3 (D) 4
(C) 3 The correct answer is (C). The word structure is /b/ /a/ /l/ . "ll" is a blend and, therefore, makes only one sound. (A) is incorrect because there is one syllable in the word "ball," not one phoneme. (B) is incorrect because "ba" makes two sounds / / b a/ / , not one. (D) is incorrect because /ll/ makes one sound, not two.
After a lesson on rounding and estimation, a teacher tells students that 157 rulers will be distributed to 4 teachers. The teacher asks the students to estimate the number of rulers each teacher will receive if the rulers are shared as equally as possible among the teachers. Which of the following students produces the best estimate for the number of rulers each teacher will receive? (A) Student A: about 30 (B) Student B: about 35 (C) Student C: about 40 (D) Student D: about 45
(C) Student C: about 40 The correct answer is (C). The question requires an understanding of how to use rounding strategies to solve problems and determine the reasonableness of results. To estimate the number of rulers each teacher will receive, one needs to estimate 157 / 4. The best estimate is produced by rounding 157 to the closest number that is easily divided by 4 in a mental calculation. Rounding 157 up to 160 yields the easy mental calculation 160 / 4, producing an estimate of 40.
Which of the following is true of qualitative measures of text complexity? (A) They describe statistical measurements of a text. (B) They rely on computer algorithms to describe text. (C) They involve attributes that can be measured only by human readers. (D) They account for the different motivational levels readers bring to texts.
(C) They involve attributes that can be measured only by human readers. The correct answer is (C). The qualitative attributes are subjective and can only be evaluated by a human reader (i.e., "predictability of text"). (A) and (B) are incorrect because they refer to quantitative attributes of text complexity, while (D) focuses on matching the reader to text and task.
x: 4, 7, 12, 15 y: -20, -38, -68, -86 Which of the following equations gives the rule for the table shown ? (A) y = -8x + 4 (B) y = -7x + 4 (C) y = -6x + 4 (D) y = -5x + 4
(C) y = -6x + 4 The correct answer is (C). The question requires an understanding of how to identify relationships between the corresponding terms of two numerical patterns. The slope of the equation can be found by calculating the rate of change for any two pairs (x, y). For example: -38 - (-20) / 7 - 4 = -6
Manuel is the tallest of the two boys. Which of the following statements about the sentence is true? (A) The sentence is written correctly. (B) The subject and verb do not agree. (C) The word "boys" should be possessive. (D) "Tallest" modifies Manuel incorrectly.
(D) "Tallest" modifies Manuel incorrectly. The correct answer is (D). "Tallest" is in the superlative degree, which is used when comparing more than two things. "Taller" is the correct word to use, since it is in the comparative degree, and therefore, the sentence is not written correctly (A). Both Manuel and "is" are singular, therefore (B) is incorrect, and (C) is incorrect because "boys" is written correctly as a plural rather than a possessive.
To make fruit punch, Edie mixes two kinds of juices in the following ratio: 1 cup of blueberry to 3 cups of red raspberry. How many cups of red raspberry will Edie need to make 48 cups of fruit punch? (A) 12 (B) 16 (C) 24 (D) 36
(D) 36 The correct answer is (D). The question requires an understanding of how to use proportional relationships to solve ratio problems. With 1 cup of blueberry juice and 3 cups of red raspberry juice, Edie can make 4 cups of punch. Since 48 / 4 = 12, to make 48 cups of punch, Edie will need 12 cups of blueberry juice and 36 cups of red raspberry juice.
Which of the following is an example of the commutative property of addition? (A) 5 x 3 = 3 x 5 (B) (1 + 7) + 4 = 1 + (7 + 4) (C) 6 x (4 + 2) = (6 x 4) + (6 x 2) (D) 8 + 9 = 9 + 8
(D) 8 + 9 = 9 + 8 The correct answer is (D). The question requires an understanding of the properties of operations. A binary operation (S △) is commutative if for any pair (a, b) where a and b are in S, a △ b = b △ a. In this case S is the set of real numbers and △ is +.
Which of the following best describes the polygon above? (A) A regular hexagon (B) An arrow (C) A convex hexagon (D) A concave hexagon
(D) A concave hexagon The correct answer is (D). The question requires an understanding of how to use attributes to classify or draw polygons. A polygon with six sides is called a hexagon. A concave polygon is a simple polygon (one whose sides do not intersect, with at least one interior angle greater than 180 degrees). A concave polygon has at least one diagonal with points outside the polygon.
Which of the following is considered a reliable source for research? (A) A wiki encyclopedia entry (B) An online discussion forum (C) A famous scientist's personal blog (D) An educational institution's Web site
(D) An educational institution's Web site The correct answer is (D). An educational institution's Web site is likely to be unbiased and contain accurate information. Wiki encyclopedia entries (A) are open to public edits and therefore are unreliable as the information cannot be verified. An online discussion forum (B) would consist of participants' opinions, and a famous scientist's personal blog (C), while it may contain scientific facts, may also contain personal opinions or biases.
Which of the following statements best describes how graphic novels promote inferencing? (A) Readers rely on characters' dialogue to tell the story. (B) Readers are given graphic organizers to facilitate understanding. (C) Readers can summarize the stories' beginning, middle, and end. (D) Readers use the pictures to interpret the text.
(D) Readers use the pictures to interpret the text. The correct answer is (D). The images in a graphic novel provide information not included within the text, such as character attributes. In a graphic novel, the combination of text and images is required to produce the complete story. (A) is incorrect because readers do not have to rely only on dialogue to understand the story. (B) is incorrect because the "graphic" in graphic novels refers to images, not to graphic organizers. (C) is incorrect because the ease of summarization is not unique to graphic novels.
Original Sentence: The sailor viewed the sky to determine how the trip would go. Revision: The mariner surveyed the clouds to predict the trip's success. The revised sentence reflects an improvement in of which of the following aspects of writing? (A) Conventions (B) Organization (C) Sentence fluency (D) Word choice
(D) Word choice The correct answer is (D). Sentence quality is enhanced by choosing words that provide specificity and clarity of meaning. The sentences both utilize the same (A) conventions and (B) organization, and while the revision sounds better, the original sentence also has (C) sentence fluency.
The Reconquista
(Or reconquest) refers to the 800 years of violence and expulsion of Muslims from the Iberian Peninsula after the failed Crusades
*Reading and Language Arts*
*Test (5002)*
*Mathematics*
*Test (5003)*
*Social Studies*
*Test (5004)*
Early abolition
- Abolitionism was a social reform effort to abolish slavery in the United States. It started in the mid-eighteenth century and lasted until 1865, when slavery was officially outlawed after the passage of the Thirteenth Amendment to the Constitution. - The movement evolved from religious roots to become a political effort that at times erupted into violence. - Though most abolitionists were white, devoutly religious men and women, some of the most powerful and influential members of the movement were African American women and men who escaped from bondage.
Causes of European migration
- After 1492, the motivations for European migration to the Americas centered around the three G's: God, gold, and glory. - Gold refers to the desire to extract natural resources like gold and sugar from the New World. - European colonizers also had a desire to spread Christianity to the New World. - Glory refers to the desire for European colonizers to increase their nation's status as a world power and gain military strength through colonization.
Jim Crow
- After Reconstruction, states in the South passed laws that barred African Americans from voting and segregated schools, restaurants, and public accommodations. - Jim Crow laws were laws created by white southerners to enforce racial segregation across the South from the 1870s through the 1960s. - Under the Jim Crow system, "whites only" and "colored" signs proliferated across the South at water fountains, restrooms, bus waiting areas, movie theaters, swimming pools, and public schools. African Americans who dared to challenge segregation faced arrest or violent reprisal. - In 1896, the Supreme Court declared Jim Crow segregation legal in the Plessy v. Ferguson decision. The Court ruled that "separate but equal" accommodations for African Americans were permitted under the Constitution.
The League of Nations
- After World War I, US President Woodrow Wilson helped to build an international peacekeeping organization. - The League of Nations was established at the end of World War I as an international peacekeeping organization. - Although US President Woodrow Wilson was an enthusiastic proponent of the League, the United States did not officially join the League of Nations due to opposition from isolationists in Congress. - The League of Nations effectively resolved some international conflicts but failed to prevent the outbreak of the Second World War.
Bleeding Kansas
- After the Kansas-Nebraska Act reopened the possibility of slavery extending into new territories, tensions between pro- and anti-slavery advocates erupted into violence. - Radical abolitionists, like John Brown, attacked and murdered white southerners in protest. A pro-slavery US Senator, Preston Brooks, viciously beat abolitionist Senator Charles Sumner on the floor of the Senate. - Bleeding Kansas foreshadowed the violence that would ensue over the future of slavery during the Civil War.
Competition between European powers
- After tobacco prices started to remain stagnant, plantation owners in Virginia wanted to move west into the Ohio River Valley to produce more tobacco and hopefully drive prices of that cash crop up. - However, both Britain and France claimed ownership of the lands in the Ohio River Valley. - The conflict between the two lead to the start of the Seven Years' War.
Labor battles in the Gilded Age
- As the United States became a major industrial power, conflict between workers and factory owners intensified. Read about the Homestead Strike and the Pullman Strike, two of the most famous labor battles in American history. - As the United States' industrial economy grew in the late 1800s, conflict between workers and factory owners became increasingly frequent and sometimes led to violence. - The Homestead Strike occurred at the Carnegie Steel Company's Homestead Steel Works in 1892. The strike culminated in a gun battle between unionized steelworkers and a group of men hired by the company to break the strike. The steelworkers ultimately lost the strike. - The Pullman Strike of 1894 started outside Chicago at the Pullman sleeping car manufacturing company and quickly grew into a national railroad strike involving the American Railway Union, the Pullman Company, railroads across the nation, and the federal government.
The Constitutional Convention
- Between May and September 1787, delegates from 12 states convened in Philadelphia to revise the Articles of Confederation, which had proven insufficient to cope with the challenges facing the young nation. - The convention was the site of spirited debate over the size, scope, and structure of the federal government, and its result was the United States Constitution. - The notorious Three-Fifths Compromise apportioned representation to the southern slaveholding states in a scheme that counted five enslaved men and women as three.
The Boston Massacre Overview
- Boston, Massachusetts was a hotbed of radical revolutionary thought and activity leading up to 1770. - In March 1770, British soldiers stationed in Boston opened fire on a crowd, killing five townspeople and infuriating locals. - What became known as the Boston Massacre intensified anti-British sentiment and proved a pivotal event leading up to the American Revolution.
Cooperation with Natives
- Both the French and the British cooperated with Native Americans to use their forces in the Seven Years' War. - The French allied with the Hurons, whereas the British allied with the Iroquois. - These alliances caused tension after the war in which the British began siding with Native Americans who were against colonial interest to move westward.
The Ghost Dance and Wounded Knee
- By 1900, there were fewer than 250,000 remaining Native Americans. - By the end of the nineteenth century, due to a series of forced removals and brutal massacres at the hands of white settlers and the US Army, the native population of North America had dwindled to a mere fraction of what it had once been. - Because forced assimilation had nearly destroyed Native American culture, some tribal leaders attempted to reassert their sovereignty and invent new spiritual traditions. The most significant of these was the Ghost Dance, pioneered by Wovoka, a shaman of the Northern Paiute tribe. - The massacre at Wounded Knee, during which soldiers of the US Army 7th Cavalry Regiment indiscriminately slaughtered hundreds of Sioux men, women, and children, marked the definitive end of Indian resistance to the encroachments of white settlers.
Overview: Environmental and health effects of European contact with the New World
- Colonization ruptured many ecosystems, bringing in new organisms while eliminating others. - The Europeans brought many diseases with them that decimated Native American populations. - Colonists and Native Americans alike looked to new plants as possible medicinal resources.
Seven Years' War: lesson overview
- Conflict between the French and the English over territory, led to a conflict known as the Seven Years' War. - The Seven Years' War solidified Britain's stance as the most dominant European country in the world. - However, as Britain attempted to increase control on the American colonies, colonists began rebelling, eventually leading to the Revolutionary War.
The Freedmen's Bureau
- Congress created the Freedmen's Bureau to economically and politically empower freed people after the Civil War. - The Freedmen's Bureau was established in March of 1865 to help freed people achieve economic stability and secure political freedoms. - Many white Southerners, as well as President Andrew Johnson, challenged the Bureau's legitimacy, sparking racial violence in the South and the ultimate failure of the Bureau. - The Bureau presented questions about the role of the federal government in establishing and maintaining racial and economic equality in the United States.
The New South
- Could the American South be remade as an industrial economy like the North? - Proponents of the New South envisioned a post-Reconstruction southern economy modeled on the North's embrace of the Industrial Revolution. - Henry W. Grady, a newspaper editor in Atlanta, Georgia, coined the phrase the "New South" in 1874. He urged the South to abandon its longstanding agrarian economy for a modern economy grounded in factories, mines, and mills. - Although textile mills and tobacco factories emerged in the South during this time, the plans for a New South largely failed. By 1900, per-capita income in the South was forty percent less than the national average, and rural poverty persisted across much of the South well into the twentieth century.
Development of the middle class
- During the Gilded Age, an expanded middle class and a wealthy upper class found new things to do with their time and money: leisure, consumption, and philanthropy. - During the Gilded Age, male and female office workers expanded the ranks of the middle class. - Larger incomes and increased leisure time among middle-class workers fostered a culture of consumption and popular amusements in American cities. - The wealthiest Americans debated whether and how to use their fortunes to improve society. In the "Gospel of Wealth," Andrew Carnegie promoted the idea that, during their lifetimes, the rich should give away their money to benefit the public.
Laissez-faire policies in the Gilded Age
- During the Gilded Age, proponents of laissez-faire policies opposed government intervention in society or the market. - Laissez-faire ideology influenced government policies toward labor relations and Reconstruction.
Technological innovation
- European colonization of the Americas was made substantially easier through several technological innovations like compasses, caravels, and astrolabes. - It affected economic development by making it possible for large scale trade networks between the Old World and the New World to develop.
Overview: motivation for European conquest of the New World
- Historians generally recognize three motives for European exploration and colonization in the New World: God, gold, and glory. - Religious motivations can be traced all the way back to the Crusades, the series of religious wars between the 11th and 15th centuries during which European Christians sought to claim Jerusalem as an exclusively Christian space. - Europeans also searched for optimal trade routes to lucrative Asian markets and hoped to gain global recognition for their country.
Shays's Rebellion
- In August 1786, Revolutionary War veteran Daniel Shays led an armed rebellion in Springfield, Massachusetts to protest what he perceived as the unjust economic policies and political corruption of the Massachusetts state legislature. - Shays's Rebellion exposed the weakness of the government under the Articles of Confederation and led many—including George Washington—to call for strengthening the federal government in order to put down future uprisings. - The rebellion, which revived the rhetoric of the American revolution, shaped debate over the proper scope and authority of the US government that ultimately resulted in the creation of the US Constitution.
Development of labor systems
- In order to extract natural resources from the Americas, European colonizers created labor systems, like the encomienda system, to exploit Native American labor. - When Native Americans began to die from diseases like smallpox, the Spanish and Portuguese began capturing and sending enslaved Africans to the Americas as a labor force.
Overview: The Spanish conquistadores and colonial empire
- In the European race to colonial dominance, the Treaty of Tordesillas legitimized Spain's holdings in the New World, indicating Spanish primacy over Portugal. - The successes of Columbus ushered in an era of Spanish conquest that led numerous other European explorers to attempt similar colonization projects. - Spain gained immense wealth from this expansionism, which translated into an influx of Spanish art and cultural capital.
Life for enslaved men and women
- In the early 19th century, most enslaved men and women worked on large agricultural plantations as house servants or field hands. - Life for enslaved men and women was brutal; they were subject to repression, harsh punishments, and strict racial policing. - Enslaved people adopted a variety of mechanisms to cope with the degrading realities of life on the plantation. They resisted slavery through everyday acts, while also occasionally plotting larger-scale revolts. - Enslaved men and women created their own unique religious culture in the US South, combining elements of Christianity and West African traditions and spiritual beliefs.
Dred Scott, the Lincoln-Douglas debates, and the election of 1860
- In the few years prior to the Civil War, an infamous Supreme Court decision, close Senate race, and monumental presidential election defined the terms for the imminent national conflict. - In 1857, the Supreme Court ruled that Black people could never be citizens of the United States in the Dred Scott v. Sandford case. - The Dred Scott decision further heightened tensions between the North and the South, and became a central issue within Abraham Lincoln and Stephen Douglas's contest for an Illinois Senate seat. - Although Douglas ultimately won the Senate race, the Lincoln-Douglas debates put Abraham Lincoln in the national spotlight, leading to his nomination for president in the election of 1860.
The Populists
- In the late nineteenth century, a new American political party sprung up to defend the interests of farmers. - The Populists were an agrarian-based political movement aimed at improving conditions for the country's farmers and agrarian workers. The Populist movement was preceded by the Farmer's Alliance and the Grange. - The People's Party was a political party founded in 1891 by leaders of the Populist movement. It fielded a candidate in the US presidential election of 1892 and garnered 8.5% of the popular vote, which was a substantial amount of support for a third party. - The Populists allied with the labor movement and were folded into the Democratic Party in 1896, though a small remnant of the People's Party continued to exist until it was formally disbanded in 1908.
The Knights of Labor
- In the late nineteenth century, the Knights of Labor attempted to organize workers of all kinds into a union to improve working hours and conditions for laborers. - Labor unions arose in the nineteenth century as increasing numbers of Americans took jobs in factories, mines, and mills in the growing industrial economy. - The Knights of Labor, founded in 1869, was the first major labor organization in the United States. The Knights organized unskilled and skilled workers, campaigned for an eight hour workday, and aspired to form a cooperative society in which laborers owned the industries in which they worked. - The Knights' membership collapsed following the 1886 Haymarket Square riot in Chicago. By 1886 the American Federation of Labor (AFL), an alliance of skilled workers' trade unions, was growing.
The Indian Wars and the Battle of the Little Bighorn
- In the late nineteenth century, the US Army clashed with Native Americans, and General Custer took his last stand. - The Indian Wars were a protracted series of conflicts between Native American Indians and white settlers over land and natural resources in the West. - Many of these battles resulted from Indian resistance to the imposition of the reservation system and the repeated attempts of the US Army and white settlers to forcibly remove Native Americans from their tribal lands. - The Battle of the Little Bighorn, also known as Custer's Last Stand, marked the beginning of the end of the Indian Wars.
Westward expansion: economic development
- In the late nineteenth century, the West developed into a modern agricultural machine--at the expense of farmers. - Land, mining, and improved transportation by rail brought settlers to the American West during the Gilded Age. - New agricultural machinery allowed farmers to increase crop yields with less labor, but falling prices and rising expenses left them in debt. - Farmers began to organize in local and regional cooperatives like the Grange and the Farmers' Alliance to promote their interests.
The reservation system
- In the nineteenth century, Native Americans were confined to reservations to open up land for white settlers. - The Indian reservation system was created to keep Native Americans off of lands that European Americans wished to settle. - The reservation system allowed indigenous people to govern themselves and to maintain some of their cultural and social traditions. - The Dawes Act of 1887 destroyed the reservation system by subdividing tribal lands into individual plots.
The Intolerable Acts and the First Continental Congress (Overview)
- In the spring of 1774, the British Parliament passed the Coercive Acts, which quickly became known in the North American colonies as the Intolerable Acts. - The Intolerable Acts were aimed at isolating Boston, the seat of the most radical anti-British sentiment, from the other colonies. - Colonists responded to the Intolerable Acts with a show of unity, convening the First Continental Congress to discuss and negotiate a unified approach to the British.
Cultural exchanges and trade networks
- Initial contact between Native Americans and European colonizers began a process of cultural and biological exchanges between the Old World and the New known as the Columbian Exchange. - The Columbian Exchange caused population growth in Europe by bringing new crops from the Americas and started Europe's economic shift towards capitalism. - Colonization disrupted ecosytems, bringing in new organisms like pigs, while completely eliminating others like beavers.
Which of the following is equivalent to 180 centimeters?
1.8 meters In 1 meter, there are 100 centimeters.
The presidency of John Adams
- John Adams, a Federalist, was the second president of the United States. He served from 1797-1801. - John Adams's presidency was marked by conflicts between the two newly-formed political parties: the Federalists and the Democratic-Republicans. - The conflicts between the two political parties centered on foreign policy and the balance of power between the federal government and the states' governments.
Chinese immigrants and Mexican Americans in the age of westward expansion
- Like Native Americans, Mexican Americans and Chinese immigrants suffered harsh consequences due to relentless westward expansion by whites in the nineteenth century. - In the nineteenth century, Mexican American, Chinese, and white populations of the United States collided as white people moved farther west in search of land and riches. - Neither Chinese immigrants nor Mexican Americans could withstand the assault on their rights by the tide of white settlers. Ultimately, both ethnic groups retreated into urban enclaves, where their language and traditions could survive. - Las Gorras Blancas, the White Caps, were a rebel group of Mexican Americans who fought back against the appropriation of their land by white settlers; in 1889 and 1890, they burned farms, homes, and crops.
The Emancipation Proclamation
- Lincoln wrote the first draft of the Emancipation Proclamation while staying with his family at the Soldier's Home, a cottage on the outskirts of Washington D.C. where they could get away from the heat of the city in summer. - He presented the Emancipation Proclamation to his cabinet on July 22, 1862 and asked for their opinions. - They approved, but Lincoln's secretary of war Edwin Stanton suggested that they wait for a big military victory to issue the proclamation so that it wouldn't seem like a desperate measure. - The summer went poorly for the US army. Not until September 17, 1862, did they win a decisive victory at the Battle of Antietam in Maryland.
Overview: The Columbian Exchange
- Mercantilism, an economic theory that rejected free trade and promoted government regulation of the economy for the purpose of enhancing state power, defined the economic policy of European colonizing countries. - Christopher Columbus introduced horses, sugar plants, and disease to the New World, while facilitating the introduction of New World commodities like sugar, tobacco, chocolate, and potatoes to the Old World. - The process by which commodities, people, and diseases crossed the Atlantic is known as the Columbian Exchange.
Juneteenth
- On June 19, 1865, the last enslaved people in the South received word from the US Army of the Emancipation Proclamation, which ordered the abolition of slavery in the South. - Black people across the country celebrate Juneteenth (a portmanteau of the words "June" and "nineteenth") to remember their history, celebrate their freedom, and renew the fight for their rights today. - In 2021, US Congress passed a bill making Juneteenth a federal holiday.
Politics in the Gilded Age
- Politics in the Gilded Age were characterized by scandal and corruption, but voter turnout reached an all-time high. - The Republican Party supported business and industry with a protective tariff and hard money policies. - The Democratic Party opposed the tariff and eventually adopted the free silver platform. - The People's (Populist) Party emerged in the 1890s to champion the interests of farmers. The party endorsed the coinage of silver to improve the financial situation of debtors.
Pontiac's uprising Overview
- Pontiac was a leader of the Odawa tribe located in the area of modern-day Ontario, Canada, and the Great Lakes region. - He led a rebellion against the British colonists after they expanded their military presence in the Great Lakes area during and after the French and Indian War. - Pontiac's uprising demonstrated the viability of pantribal cooperation in the struggle against European-American territorial expansionism and contributed to the deterioration of relations between Great Britain and its North American colonies.
Black Codes
- Southern states enacted black codes after the Civil War to prevent African Americans from achieving political and economic autonomy. - When slavery was abolished at the end of the Civil War, southern states created black codes, laws which aimed to keep white supremacy in place. - Black codes attempted to economically disable freed slaves, forcing African Americans to continue to work on plantations and to remain subject to racial hierarchy within the southern society. - Black codes gave rise to a new wave of radical Republicanism in Congress, and the eventual move towards enshrining racial equality into the Constitution. However, black codes also set precedent for Jim Crow laws.
Motivations for colonization
- Spain's colonization goals were to extract gold and silver from the Americas, to stimulate the Spanish economy and make Spain a more powerful country. - Spain also aimed to convert Native Americans to Christianity.
Interactions with Native Americans
- Spanish colonizers attempted to integrate Native Americans into Spanish culture by marrying them and converting them to Catholicism. - Although some Native Americans adopted aspects of Spanish culture, others decided to rebel. - The Pueblo Revolt was one example of a successful Native American effort to reclaim their religious practices, culture, and land.
The Gold Rush
- The 1848 discovery of gold in the territory of California prompted 300,000 hopeful prospectors to flood into the region, altering it forever. - The 1848 discovery of gold in California set off a frenzied Gold Rush to the state the next year as hopeful prospectors, called "forty-niners," poured into the state. - This massive migration to California transformed the state's landscape and population. - The Gold Rush was characterized by violent clashes among settlers, miners, and Native Americans over access to the land and its natural resources.
The Dawes Act
- The 1887 law intended to assimilate Native Americans led to the loss of millions of acres of land. - The Dawes Act of 1887 authorized the federal government to break up tribal lands by partitioning them into individual plots. Only those Native Americans who accepted the individual allotments were allowed to become US citizens. - The objective of the Dawes Act was to assimilate Native Americans into mainstream US society by annihilating their cultural and social traditions. - As a result of the Dawes Act, over ninety million acres of tribal land were stripped from Native Americans and sold to non-natives.
The Treaty of Versailles
- The 1919 Treaty of Paris ended World War I, but imposed heavy penalties on Germany. - The Treaty of Versailles was signed on June 28, 1919, and officially ended the war between Germany and the Allied Powers. - The controversial War Guilt clause blamed Germany for World War I and imposed heavy debt payments on Germany. - The Treaty of Versailles was a major contributing factor in the outbreak of the Second World War.
Lexington and Concord
- The Battles of Lexington and Concord, fought on April 19, 1775, were the first military clashes of the American Revolutionary War. - The Massachusetts militia routed the British Army forces and were soon joined by militias from Connecticut, New Hampshire, and Rhode Island. These militias would become the core of the Continental Army.
The Bill of Rights
- The Bill of Rights is the name given to the first 10 amendments to the US Constitution. - The Bill of Rights consists of guarantees of civil liberties and checks on state power; it was added in order to convince states to ratify the Constitution.
The Boston Tea Party (Overview)
- The Boston Tea Party, which involved the willful destruction of 342 crates of British tea, proved a significant development on the path to the American Revolution. - The Boston Tea Party, which occurred on December 16, 1773 and was known to contemporaries as the Destruction of the Tea, was a direct response to British taxation policies in the North American colonies. - The British response to the Boston Tea Party was to impose even more stringent policies on the Massachusetts colony. The Coercive Acts levied fines for the destroyed tea, sent British troops to Boston, and rewrote the colonial charter of Massachusetts, giving broadly expanded powers to the royally appointed governor.
The Compromise of 1850
- The Compromise of 1850 acted as a temporary truce on the issue of slavery, primarily addressing the status of newly acquired territory after the Mexican-American War. - Under the Compromise, California was admitted to the Union as a free state; the slave trade was outlawed in Washington, D.C., a strict new Fugitive Slave Act compelled citizens of free states to assist in capturing enslaved people; and the new territories of Utah and New Mexico would permit white residents to decide whether to allow slavery. - Ultimately, the Compromise did not resolve the issue of slavery's expansion; instead, the fiery rhetoric surrounding the Compromise further polarized the North and the South.
The Compromise of 1877
- The Compromise of 1877 gave Rutherford B. Hayes the presidency in exchange for the end of Reconstruction in the South. - The Compromise of 1877 resolved the disputed 1876 presidential election between Democratic candidate Samuel Tilden and Republican candidate Rutherford B. Hayes. - Democrats agreed that Rutherford B. Hayes would become president in exchange for the withdrawal of federal troops from the South and the granting of home rule in the South. - President Hayes' withdrawal of federal troops from Louisiana and South Carolina marked a major turning point in American political history, effectively ending the Reconstruction Era and issuing in the system of Jim Crow.
The Federalist Papers
- The Federalist Papers was a collection of essays written by John Jay, James Madison, and Alexander Hamilton in 1788. - The essays urged the ratification of the United States Constitution, which had been debated and drafted at the Constitutional Convention in Philadelphia in 1787. - The Federalist Papers is considered one of the most significant American contributions to the field of political philosophy and theory and is still widely considered to be the most authoritative source for determining the original intent of the framers of the US Constitution.
The Homestead Act and the exodusters
- The Homestead Act of 1862 gave free land to Americans willing to improve it, regardless of race, sex, or nation of origin. - The Homestead Act of 1862 parceled out millions of acres of land to settlers. All US citizens, including women, African Americans, freed slaves, and immigrants, were eligible to apply to the federal government for a "homestead," or 160-acre plot of land. - Homesteading was a contentious issue, because Northerners and Republicans wanted to open the land to settlement by individual farmers, while Southern Democrats sought to make the land available only to slaveholders. - The exodusters were African American migrants who left the South after the Civil War to settle in the states of Colorado, Kansas, and Oklahoma.
The Kansas-Nebraska Act and party realignment
- The Kansas-Nebraska Act organized two new territories in the land acquired through the Louisiana Purchase, Kansas and Nebraska. The act established that in these territories, the principle of popular sovereignty would apply, meaning that the white residents of each territory would vote on whether to permit slavery when applying for statehood. - The Act repealed the Missouri Compromise of 1820, which drew the horizontal line of slavery across the West along the 36° 30' parallel, as both Kansas and Nebraska were north of this line. This reopened the question of slavery's western expansion. - The passage of the Kansas-Nebraska Act induced party realignment and violence, furthering the sectional divide that ultimately erupted in the Civil War.
The First KKK
- The Ku Klux Klan, a terror organization, gained political footing during Reconstruction in the postbellum South. - The Ku Klux Klan was a white supremacist terrorist group that emerged during Reconstruction. It took violent steps to undermine the Republican party, hoping to maintain black economic instability and ensure white racial and economic superiority in the postwar South. - Congress countered the KKK with the Force Acts and the Ku Klux Klan Act of 1871, which made Klan violence and political intimidation illegal under federal law. - Republicans lost hold within the South largely due to Klan violence, allowing the South to maintain a ruling racial order that morphed into Jim Crow.
Overview: Pueblo uprising of 1680
- The Pueblo people, Native Americans living in what is now New Mexico, rose up against Spanish conquistadores in the wake of religious persecution, violence, and drought. - The uprising aimed to reclaim Pueblo religious practices, culture, and land, which had been stripped away by Spanish conquistadores. - Although the Pueblo uprising ultimately failed to take back Santa Fe from Spanish colonizers, the Pueblo people made a lasting impact on the dominant culture of the Southwest.
The Second Continental Congress
- The Second Continental Congress convened in Philadelphia in the summer of 1775, shortly after the war with the British had begun. It was preceded by the First Continental Congress in the fall of 1774. - The Congress appointed George Washington as commander of the Continental Army, and authorized the raising of the army through conscription. - On July 4, 1776, the Congress issued the Declaration of Independence, which for the first time asserted the colonies' intention to be fully independent of the mother country. - The Congress established itself as the central governing authority under the Articles of Confederation, which remained in force until 1788.
Uproar over the Stamp Act Overview
- The Stamp Act was enacted in 1765 by British Parliament. It imposed a direct tax on all printed material in the North American colonies. - The most politically active segments of colonial society—printers, publishers, and lawyers—were the most negatively affected by the act. - The Stamp Act intensified colonial hostility toward the British and was a pivotal development on the road to the American Revolution.
Life after slavery for African Americans
- The Thirteenth Amendment (1865) ended slavery, and slavery's end meant newfound freedom for African Americans. - During the period of Reconstruction, some 2000 African Americans held government jobs. - The black family, the black church, and education were central elements in the lives of post-emancipation African Americans. - Many African Americans lived in desperate rural poverty across the South in the decades following the Civil War.
The Townshend Acts and the committees of correspondence Overview
- The Townshend Acts, passed in 1767 and 1768, were designed to raise revenue for the British Empire by taxing its North American colonies. They were met with widespread protest in the colonies, especially among merchants in Boston. - The Townshend Acts renewed a fierce debate over the British Parliament's right to tax the colonies. - In 1772, Boston revolutionary Samuel Adams urged the creation of a committee of correspondence to communicate with other colonial assemblies, educate townspeople about their political rights, and rally opposition to British rule.
Social Darwinism in the Gilded Age
- The belief that white, wealthy, Anglo-Saxon Americans were biologically superior to other groups fueled many social and political trends of the Gilded Age. - Social Darwinism is a term scholars use to describe the practice of misapplying the biological evolutionary language of Charles Darwin to politics, the economy, and society. - Many Social Darwinists embraced laissez-faire capitalism and racism. They believed that government should not interfere in the "survival of the fittest" by helping the poor, and promoted the idea that some races are biologically superior to others. - The ideas of Social Darwinism pervaded many aspects of American society in the Gilded Age, including policies that affected immigration, imperialism, and public health.
America moves to the city
- The industrial boom of the late nineteenth century led Americans and immigrants from the world over to leave farming life and head to the city. - Americans increasingly moved into cities over the course of the late nineteenth and early twentieth centuries, a movement motivated in large measure by industrialization. - Eleven million people migrated from rural to urban areas between 1870 and 1920, and a majority of the twenty-five million immigrants who came to the United States in these same years moved into the nation's cities. - By 1920, more Americans lived in cities than in rural areas for the first time in US history.
The presidency of George Washington
- Virginian and Revolutionary War General George Washington became the United States's first president in 1789. His actions in office set a precedent for a strong executive branch and a strong central government. - The major political questions and conflicts during the 1790s concerned foreign policy, economic policy, and the balance of power between states and the federal government. - During Washington's presidency, factions began to emerge that would soon form the first two political parties in the United States: the Democratic-Republicans and the Federalists. - Washington's decision to stay neutral during the French Revolution set a precedent for the United States government to practice isolationism as its main foreign policy strategy for over a hundred years.
Migration after the war
- When the Seven Years' War ended, the British won all of France's land holdings in colonial America. - Colonists wanted to expand westward into these new lands in order to gain more land, but fearing conflicts with Native Americans, Britain passed the Proclamation of 1763. - The proclamation prohibited movement west of the Appalachian Mountains, upsetting many colonists who wanted the land to increase their wealth. - Thousands of colonists defied the law, moving west to claim land for themselves.
Gilded Age politics: patronage
- Widespread corruption in the Gilded Age only gave way to civil service reform after President James Garfield was assassinated. - During the Gilded Age, politics were riddled with corruption as presidents awarded government positions to political supporters through the patronage or spoils system. - Although several presidents made limited efforts toward reforming the spoils system, it was not until disappointed office-seeker Charles Guiteau assassinated president James Garfield in 1881 that civil service reform garnered widespread support. - The Pendleton Civil Service Act was the first significant piece of anti-patronage legislation. The act created the Civil Service Commission to regulate and limit patronage positions.
The presidency of Woodrow Wilson
- Wilson campaigned for a second term on the slogan "He kept us out of war." But that wouldn't be true for long. - Woodrow Wilson was the 28th president of the United States. He served two terms in office, from 1913 to 1921. - Wilson was a Progressive Democrat who believed in the power of the federal government to expose corruption, regulate the economy, eliminate unethical business practices, and improve the general condition of society. - During Wilson's years in office, the US federal government was segregated and the Ku Klux Klan experienced a major revival. - Wilson's second term in office was dominated by the First World War. Though Wilson campaigned on the slogan "He kept us out of war," escalating German aggression ultimately made it impossible for the United States to stay out of the conflict.
The slave economy
- With the invention of the cotton gin, cotton became the cash crop of the Deep South, stimulating increased demand for enslaved people from the Upper South to toil the land. - As the disparity between plantation owners and poor white people widened in the Deep South, deeply entrenched racism blurred perceived class divides. - The slave economy of the South had international economic reach since the majority of cotton was sold abroad; it connected the United States to the international marketplace.
Women in the American Revolution
- Women performed crucial tasks in the American Revolution, organizing fundraising drives, supplying the troops, working in the military camps, and tending to the wounded soldiers. - One of the most common ways that women supported the war effort was by making homespun, home-made cloth that took on revolutionary symbolism after the colonies imposed boycotts on British goods, including textiles. - Some women even acted as spies, and there is at least one documented case of a woman disguising herself as a man to fight in the war.
The United States in World War I
- World War I was the "war to end all wars." It had major consequences on Americans both at home and abroad. - World War I was the deadliest conflict until that point in human history, claiming tens of millions of casualties on all sides. - Under President Woodrow Wilson, the United States remained neutral until 1917 and then entered the war on the side of the Allied powers (the United Kingdom, France, and Russia). - The experience of World War I had a major impact on US domestic politics, culture, and society. Women achieved the right to vote, while other groups of American citizens were subject to systematic repression.
For x = 4 and y = -3, what is the value of 2x + 3y?
-1
If 2x² = 50, which of the following could be the value of x?
-5 and 5
The probability that event A will NOT happen is equal to...
1 - P(A)
Two triangles are similar if they meet one of the following criteria:
1) AA or Angle-Angle: two pairs of corresponding angles are equal 2) SSS or Side-Side-Side: three pairs of corresponding sides are proportional 3) SAS or Side-Angle-Side: two pairs of corresponding sides are proportional and the corresponding angles between them are equal
When working with negative numbers, remember that...
1) Negative x negative = positive 2) Positive x negative = negative
A flowchart is a visual representation of a sequence of operations. To calculate the result given the input, we... To calculate the input given the result, we...
1) Perform the operations in order 2) Perform the inverse operations in reverse order
Two triangles are congruent if they meet one of the following criteria:
1) SSS or Side-Side-Side: all three pairs of corresponding sides are equal. 2) SAS or Side-Angle-Side: two pairs of corresponding sides and the corresponding angles between them are equal 3) ASA or Angle-Side-Angle: two pairs of corresponding angles and the corresponding sides between them are equal 4) HL or Hypotenuse-Leg: the pair of hypotenuses and another pair of corresponding sides are equal in two right triangles ASS or Angle-Side-Side DOES NOT WORK!
For a triangle with only two known side lengths (a and b) the unknown side length (x) must meet one of the following conditions:
1) Shorter than the sum of the two known side lengths 2) Longer than the positive difference of the two known side lengths
A box contains 5 red, 10 green, and 20 orange candies. If one candy is selected randomly from the box, what is the probability that the selected candy will be red?
1/7
The 22 students in Ms. Smith's 2nd grade class each have a sibling or a pet. If 14 students have a sibling and 18 students have a pet, how many students have both a sibling and a pet?
10 Solving 14 + 18 - x = 22 for x gives us 10 students who have both a sibling and a pet.
10 miles = 1 inch According to the scale shown above, which of the following represents 100 miles?
10 inches
The ratio of fiction books to non-fiction books in Roxane's library is 7 to 4. If Roxane owns 182 fiction books, how many non-fiction books does she own?
104
What is 30% of 40?
12
At Honey Beepot, the bulk price for honey is $2.50 per pound, with a minimum purchase of 20 pounds. If Bobby paid $80 for some honey, by how many pounds did Bobby's purchase exceed the minimum?
12 Bobby purchased 80/2.50 = 32 pounds of honey, which exceeds the minimum purchase by 32 - 20 = 12 pounds.
Jack had three babysitting jobs this week. He worked the same number of hours, "H," on each job. He was paid at a rate of $12 for every hour at his first job, $4 for every half hour at his second job, and $5 for every 20 minutes at his third job. Which of the following expressions could be used to find the total amount, in dollars, Jack earned?
12H + 8H + 15H The question requires an understanding of how to translate between verbal statements and algebraic expressions or equations. Jack made 12 dollars per hour at his first job. He made 4 dollars per half hour, or 4 x 2 = 8 dollars per hour, at his second job. Finally, Jack made 5 dollars for each 20 minutes, or 5 x 3 = 15 dollars per hour, at his third job. If he worked "H" hours at each job, he made 12H + 8H + 15H.
For x > 0, if x² = 169, what is the value of x?
13
When carrying a large backpack and four liters of water, Yogi weighs 225.6 pounds. If the backpack weighs 55.4 pounds and the water weighs 8.8 pounds, what is Yogi's weight without the backpack and the water?
161.4 pounds
A florist needs to make 25 centerpieces for a large wedding reception. Each centerpieces requires 7 roses. If the florist can only buy roses by the dozen, what is the least number of roses that the florist must buy?
180
A pancake recipe uses 1/4 cup of all-purpose flour and 1/4 cup of rice flour. What is the ratio of all-purpose flour to rice flour in the recipe?
1:1
Which of the following expressions are equivalent to 7x + 1?
2 (2x - 1) + 3 (x + 1) and 5 (x + 1) + 2 (x - 2)
Which of the following expressions is equivalent to 4x - 3 for all values of x?
2 (2x - 1) - 1
A circle graph (pie chart)
A circle that is divided into as many sections as there are categories of the qualitative variable, the area of each section represents, for each category, the value of the quantitative data as a fraction of the sum of values, the fractions sum to 1; sometimes the section labels include both the category and the associated value or percent value for that category
A triangle has side lengths of 10, 20, and x. Which of the following could be the value of x?
20 10 + 20 > 20
If 1/6x + 3 = 7, what is the value of x?
24
2 2 3 5 x y In the figure above, the product of the three numbers in the horizontal row is equal to the sum of the four numbers in the vertical column. What is the value of x + y?
25
Pippin owns 2 cats, 3 dogs, and a lizard as pets. What is the ratio of the number of cats to the total number of pets Pippin owns?
2:6 → 1/3
If 1/2 (6x + 8) = kx + 4, what is the value of k?
3
If x² = 9, which of the following could be the value of x?
3
When 2 is added to the product of 6 and a certain number, the result is 20. What is the value of the number?
3
When the sum of a certain number and 1 is multiplied by 3, the result is 6. Which of the following equations matches the description above?
3 (x + 1) = 6
Which of the following are factors of both 15 and 45?
3 and 5 15 ÷ 3 = 5, so 3 is a factor of 15. 45 ÷ 3 = 15, so 3 is a factor of 45. 15 ÷ 5 = 3, so 5 is also a factor of 15. 45 ÷ 5 = 9, so 5 is also a factor of 45.
Jasmine is working on an art project. She has one piece of construction paper that is 21 cm wide and a second piece that is 33 cm wide. Jasmine wants to cut both pieces of paper into strips that are equal in width and as wide as possible. How wide should Jasmine cut each strip?
3 cm
8 (1 dot) 9 (3 dots) 10 (2 dots) 11 (1 dot) Number of Hours Worked The dotplot above shows the number of hours Kim worked each weekend since she started her new job. Each dot represents a different weekend. On what fraction of the weekends did Kim work at least 10 hours?
3/7
Nicholas drinks 8 ounces of milk for every 5 cookies he eats. If he eats 20 cookies, how many ounces of milk does he drink?
32 ounces
If Balto ran 33 miles in 3 hours, then his speed was...
33 miles / 3 hours = 11 miles/hour (miles per hour)
The box for a board game is 11 inches long, 11 inches wide, and 3 inches tall. What is the volume of the box in cubic inches?
363
The radius of a circular mirror is 6 inches. What is the area, in square inches, of the mirror?
36π
At a concert, Anton bought 3 bottles of water for x dollars each and a T-shirt for 25 dollars. Which of the following represents the total amount Anton spent in dollars?
3x + 25
Eleanor bought 5 pints of frozen yogurt and a tray of jumbo shrimp from The Food Place for a total of $45. If the price of a tray of jumbo shrimp is $25, what is the price of a pint of frozen yogurt?
4
If 5 times the square of a certain number is 80, which of the following could be that number?
4
The base of a triangle is 6 inches long. If the area of the triangle is 12 square inches, what is the height of the triangle in inches?
4
A rectangle with a perimeter of 10 units has an area of at least 5 square units. Which of the following rectangles shows that the statement above is NOT correct?
4 x 1 rectangle This rectangle has a perimeter of 2(4 + 1) = 10 units and an area of 4 x 1 = 4 square units, which is smaller than 5 square units. Since it satisfies the condition but not the conclusion, this rectangle can be used as a counterexample.
Tayo got the following scores on her Spanish quizzes: 88, 96, 94, 92, 98, 58, 90. What is the range of Tayo's quiz scores?
40 98 - 58 = 40
If 3x - 5 = 35, what is the value of x?
40/3
The price of a particular model of headphones was $7 in 2016. In 2018, the price of the same model of headphones was $10. What is the approximate percent increase in the price of the headphones?
43%
If 3x + 5y = 9, what is the value of 15x + 25y?
45
If the product of a certain number and 4 is 64, which of the following equations, when solved, would give us the value of the number?
4x = 64
In a classroom, there are 7 groups of desks, and each group has the same number of desks. The math teacher distributes one calculator to each desk. If out of 37 calculators all but 2 are distributed, how many desks are in each group?
5
What is the radius, in inches, of a circle with a circumference of 10π inches?
5
The first term in a recursive sequence is 1. Each subsequent term in the sequence is calculated by multiplying the previous term by 3 and subtracting 1 from the result. Which of the following are terms in the sequence?
5 and 14 and 41 5 is the third term in the recursive sequence. 14 is the fourth term in the recursive sequence. 41 is the fifth term in the recursive sequence.
Craig bought 96 rolls or toilet paper for 48 dollars. What is the cost of a single roll of toilet paper?
50¢
Two of the lights in Sarah's homeroom are flickering. They both just flickered at the same time. One of the lights flickers every 7 seconds and the other light flickers every 8 seconds. How many seconds will pass before both lights flicker at the same time again?
56 seconds
Ellen spent 3 hours and 40 minutes driving from City A to City B and 2 hours and 50 minutes driving back from City B to City A. What is the total time she spent driving from City A to City B and back?
6 hours and 30 minutes
75, 70, 45, 50, 52, 68 A veterinary student is studying newborn giraffes. The list above shows the masses of 6 newborn giraffes, rounded to the nearest kilogram. What is the mean mass of the newborn giraffes? Round your answer to the nearest kilogram
60 kilograms
What percent of 20 is 12?
60%
A mouse's heart beats 900 times in 1.5 minutes. At this rate, how many times does the mouse's heart beat in 10 minutes?
6000 times
If x + 2 = 9, what is the value of x?
7
Input x → add 1 → multiply by 2 → subtract 5 → print result If the result printed according to the flowchart above is 11, what was the value of the input x?
7
Tara has read 95% of the books she owns. If Tara owns 160 books, how many of her books has she NOT read?
8
What is 3/5 ÷ 2/3 expressed as a single fraction?
9/10
A gumball machine contains red, blue, and green gumballs. There are 70 red gumballs and 50 blue gumballs. When the handle of the machine is turned, one gumball is randomly dispensed. If the probability of getting a red gumball is 1/3, how many green gumballs are in the machine?
90
While eating ants, a giant anteater flicks its tongue 150 times per minute. At this rate, how many times does the giant anteater flick its tongue in 6 minutes?
900 times
Total distance...
= speed × time
Total cost...
= unit price × quantity
Area of a triangle
A = 1/2bh
Area of a parallelogram
A = bh
Area of a rectangle
A = lw
Area of a square
A = s²
The formula for "A," the area of a circle with radius "r," is...
A = πr² or A = π(d/2)²
A multiple
A number that results when we multiply a whole number by another non-zero whole number
Bartolome de las Casas
A Spanish priest who criticized Spain's treatment of Native Americans in its colonies
Ratio
A comparison of two quantities
A mathematical statement is composed of two parts:
A condition and a conclusion
Pontiac's uprising (1763)
A force of 300 members of different Native American tribes led by Chief Pontiac attempted to stop British encroachment on their territory in an armed rebellion
Encomienda system
A labor system in which the Spanish crown authorized Spaniards, known as encomenderos, to enslave native people to farm and mine in the Americas
Proclamation of 1763
A law passed by the British parliament that prohibited colonial movements west of the Appalachian Mountains
Transversal
A line that intersects two or more lines
Past participle
A past tense verb that is used as an adjective, for example: "fallen leaves"
Albany Plan of Union (1754)
A plan created by Benjamin Franklin to organize an intercolonial government, including a system to collect taxes and recruit troops
A percentage
A ratio whose second term is 100
A scale factor is...
A relationship we use to translate between representation and reality, for example: each inch on a map represents a certain number of miles in the real world
Caravel
A small, fast ship used by the Spanish and Portuguese, which made exploration easier
Caste system
A social system in which class status is determined at birth; the Spanish had mixed-race children in the Americas with enslaved Africans and Native Americans; status was determined by how "Spanish" one was, so those with little to no Spanish blood were in the lowest class
Pueblo Revolt (1680)
A successful rebellion by Pueblo Native Americans to reclaim Pueblo religious practices, culture, and land that had been stripped away by Spanish colonizers
Triangle ABC (7, 7, 4) is shown above. Which of the following triangles are similar to Triangle ABC?
A triangle with side lengths of 1, 1, and 4/7. Each side of this choice is 1/7 the length of its corresponding side in triangle ABC. Therefore, it meets the SSS similarity criterion. A triangle with side lengths of 35, 35, and 20. Each side of this choice is 5 times the length of its corresponding side in triangle ABC. Therefore, it meets the SSS similarity criterion.
Center describes...
A typical value of a data point; two measures of center are mean and median
In which of the following does the digit in the thousandths place have the greatest value?
A) 1,247.5288
What is the value of 3/5 x 1/6?
A) 1/10
What is the value of 5/6 - 1/2?
A) 1/3
Historical figures such as Christopher Columbus and Noah Webster observed that changes in land cover caused changes in climate. Columbus believed that the presence of forests on the newly discovered islands of the West Indies caused them to have more rainfall than the deforested Azores and Canary Islands. Computer models of the effects of deforestation of tropical islands, analyzed 500 years after Columbus's initial observation, indicate that forested islands should have about three times as much rainfall as do equivalent deforested islands. Which of the following best describes the organization of the passage? A) A historical observation is discussed, and then evidence supporting that observation is presented. B) A historical theory is described and then revised in light of recent findings. C) A historical event is discussed, and the long-term effects of that event are analyzed. D) Observations made by two historical figures are compared, and one set of observations is shown to be more accurate. E) The assumptions behind a particular historical belief are identified and then discredited.
A) A historical observation is discussed, and then evidence supporting that observation is presented. This is the best choice. "A historical observation is discussed" summarizes the first two sentences of the passage. The final sentence of the passage provides evidence supporting Columbus's theory. The final sentence of the passage states that evidence produced by computer models supported Columbus's claim 500 years after he made it.
Whatever their disadvantage with respect to distributing education tax dollars evenly among school districts, in one respect at least, local property taxes are superior to state taxes as a means of funding public schools. Because local property taxes provide public schools with a direct source of revenue, these public schools do not have to compete with other government services for tax dollars. School administrators do not have to compete for a share of the state tax dollars, which are already being spent on health, criminal justice, public safety, and transportation. They are not placed in the position of having to argue that school programs must have priority over other public services financed by state taxes. Which of the following would be most similar to the advantage of funding schools with local property taxes as described in the passage? A) A small community holds a fundraiser to repair a neglected local road B) A high school increases its arts budget due to the contribution of a wealthy alumnus C) A government department receives increased funding due to cuts to other programs D) An educational research project is subsidized by a government grant E) Each community center in a wide region receives equal funding from a centralized source
A) A small community holds a fundraiser to repair a neglected local road This is the best choice. Just as schools funded by local property taxes find "a direct source of revenue" in the local community, this community has taken initiative to raise funds for local road repairs. In this way, the road repairs won't need to compete with other public services for government funding.
"Celebrities have a tremendous influence on the young, and for that reason, they have a responsibility to act as role models." Discuss the extent to which you agree or disagree with this point of view. Support your position with specific reasons and examples from your own experience, observations, or reading. Which of the following sentences provide specific examples relevant to this prompt? A) As basketball star Charles Barkley stated in a famous advertising campaign for Nike, he was paid to dominate on the basketball court, not to raise your kids. B) Many celebrities do consider themselves responsible for setting a good example and create non-profit organizations through which they can benefit youths. C) Many celebrities, like Kylie Jenner with her billion-dollar cosmetics company, profit directly from being imitated by fans who purchase sponsored products. D) My ten-year-old nephew may love Drake's music, but his behaviors are more similar to those of the adults he interacts with on a daily basis, like his parents and teachers. E) It's very common for young people to wear fashions similar to those of their favorite celebrities.
A) As basketball star Charles Barkley stated in a famous advertising campaign for Nike, he was paid to dominate on the basketball court, not to raise your kids. This is a specific and relevant example. It discusses a single instance in which a clearly identified individual makes an argument that relates directly to the prompt. C) Many celebrities, like Kylie Jenner with her billion-dollar cosmetics company, profit directly from being imitated by fans who purchase sponsored products. This is a specific and relevant example. It discusses a single instance in which a clearly identified celebrity profits directly from serving as a role model, which is relevant to the prompt. D) My ten-year-old nephew may love Drake's music, but his behaviors are more similar to those of the adults he interacts with on a daily basis, like his parents and teachers. This is a specific and relevant example. It discusses personal experience with a clearly identified individual, and it relates that experience directly to the prompt by showing that celebrities are less responsible for a young person's behavior than other role models.
Scientists consider both landslides and surface-creep movement essential in the formation of rock glaciers. Evidence of landslides can be distinguished from that of surface-creep movement because landslides leave a deeper and more definite surface of rupture, partly due to their faster rate of movement. Those studying the origins of rock glaciers have noted that some glaciers are well-defined, while others are not; that is, some show evidence of deep ruptures, while others do not. Given the information in the passage, which of the following may be concluded about rock glaciers? A) Not all rock glaciers originate in the same way. B) Landslides initiate the formation of rock glaciers, then surface-creep movement follows. C) Neither landslides nor surface-creep movement account for the formation of rock glaciers. D) While the definition and depth of rupture can be measured at rock glacier sites, the rate of movement cannot. E) Further study is required to determine the origins of rock glaciers.
A) Not all rock glaciers originate in the same way. This is the best choice. According to the passage, some rock glaciers show deep ruptures, while others don't. Since deep ruptures are a sign of landslide formation, it logically follows that some glaciers originate from landslides, while others don't.
In our increasing awareness of ecological health, many industrial practices have come under close examination, and mining is no exception. Though drilling is required in both cases, base-metal mining involves toxic chemical leachates for separating the metal from the rock, whereas diamond mining does not: diamonds can be separated from surrounding rock using only crushers, screens, and all-natural water. Thus, base-metal mining is environmentally destructive, but diamond mining does not harm the environment. Which of the following, if true, would most weaken the author's argument concerning the effect of diamond mining on the environment? A) The process of drilling and getting the drill rig to and from the site destroys ecological habitats. B) Base metals have utilitarian value, but diamonds are functionally almost worthless. C) Toxic chemical leachates contaminate not only soil, but groundwater as well. D) There have been proposals to use abandoned mine shafts as garbage dumps. E) Logging can be as ecologically destructive as mining.
A) The process of drilling and getting the drill rig to and from the site destroys ecological habitats. This is the best choice. The author argues that diamond mining doesn't harm the environment because it doesn't use chemical leachates. However, this choice states that diamond mining still harms the environment in other ways.
Which of the following sequence of steps, when completed, will solve the equation 6x - 7 = 8 for x?
Add 7 to both sides of the equation, then divide both sides by 6.
Supplementary angles
Add up to 180° and are usually seen on the same side of an intersection of two lines
The Crusades
Began by European Christians, a campaign of violence against Muslims to dominate the Holy Lands, partially in response to sustained Muslim control in Europe
Adjective/adverb confusion
Adjectives and adverbs can't be used interchangeably
The Fifteenth Amendment extended the right to vote to...
African American men. But this new constitutional guarantee faced serious challenges due to new laws in the South, called Black Codes, which aimed to disenfranchise freedmen.
Linear equation
An equation with variable(s) to the first power and one or more constants, for example: 2x + 3 = 4
The area of a circle is the...
Amount of flat space inside the circle's circumference
The Holy Lands
An area that extended from modern-day Turkey in the north along the Mediterranean coast to the Sinai Peninsula, under Islamic control
Secondary source
An artifact or document that relates, discusses, or analyzes information that was originally presented somewhere else. Secondary sources build on primary sources by compiling and filtering authentic information through the viewpoint of a scholar or other third party.
Primary source
An artifact, document, or any other source of information authentic to the time, person, or subject of study. Primary sources serve as the original source of information about a subject. They don't include scholarly commentary and aren't filtered through any other viewpoint.
Mercantilism
An economic theory that was designed to maximize trade for a nation and especially maximize the amount of gold and silver a country had
Proportion
An equality of two ratios
Quadratic equation
An equation with at least one variable to the second power as its highest power term and one or more constants, for example: 3x² = 48
A probability of 1/2 or 50% means that...
An event is equally likely to happen or not happen
A probability of 1 or 100% means that...
An event will certainly happen
A probability of 0 or 0% means that...
An event will never happen
Interior angles
Angles between adjacent sides inside a polygon, a triangle has three of them; the measures of the three interior angles in a triangle add up to 180°
the Columbian Exchange
As Europeans traversed the Atlantic, they brought with them plants, animals, and diseases that changed lives and landscapes on both sides of the ocean; two-way exchanges between the Americas and Europe/Africa
Mean
Average (to find, add up all numbers together than divide by the amount of numbers you have) Useful for describing the center of data with similar values, the average value Mean = sum of values/number of values
What is 5/6 of 0.7?
B) 7/12 5/6 x 7/10 = 35/60, which simplifies to 7/12.
"Celebrities have a tremendous influence on the young, and for that reason, they have a responsibility to act as role models." Discuss the extent to which you agree or disagree with this point of view. Support your position with specific reasons and examples from your own experience, observations, or reading. Which of the following sentences could serve as strong thesis statements for an essay responding to this prompt? A) Kids should find role models that are worthier than celebrities because celebrities may be famous for reasons that aren't admirable. B) Because they profit from the admiration of youths, celebrities have a moral responsibility for the reactions their behaviors provoke in fans. C) Celebrities may have more imitators than most people, but they hold no more responsibility over the example they set than the average person. D) Notoriety is not always a choice, and some celebrities may not want to be role models. E) Parents have a moral responsibility to serve as immediate role models for their children.
B) Because they profit from the admiration of youths, celebrities have a moral responsibility for the reactions their behaviors provoke in fans. This is a strong thesis. This sentence responds directly to the prompt by stating that celebrities are responsible for setting a good example for the young people who admire them. C) Celebrities may have more imitators than most people, but they hold no more responsibility over the example they set than the average person. This is a strong thesis. It responds directly to the prompt by stating that celebrities are no more responsible for being good role models than anyone else, despite their increased visibility.
Histories of the Middle East abound in stereotypes and clichés, particularly with respect to women. The position of women in the Middle East is frequently treated as though Middle Eastern societies formed a single unit that could be accurately represented in a simple description. The author of the passage suggests which of the following about histories of the Middle East with regard to their treatment of women? A) A general problem with such histories was first noticed in their descriptions of the role of women. B) The experience of women in Middle Eastern societies is much more diverse than such histories have assumed. C) The study of women's roles and experience has recently become a central focus in such histories. D) Such histories report that the position of women in Middle Eastern societies has undergone a major transformation. E) Until recently, such histories typically neglected to discuss the position of women.
B) The experience of women in Middle Eastern societies is much more diverse than such histories have assumed. This is the best choice. The author speaks negatively of these histories, stating that they're based on stereotypes. Specifically, the author criticizes the view that women from all Middle Eastern societies share a single role that can be easily described. This criticism implies the author believes the opposite: on the contrary, women's roles in the Middle East are numerous and complex.
Despite the prevalence of digital books, some researchers have advocated for the continued use of print books over e-readers. Some studies link late-night reading on screens to overall poor health, including lack of sleep and disturbance of circadian rhythms. Other studies have suggested that the human brain may not retain information from a digital screen as well as information read on paper. But digital books have some major advantages over print books: ease of acquisition and portability. Isn't access to books more important than the medium in which people read? Furthermore, the portability of digital books makes it easier to snatch extra moments of reading throughout the day. The average person wouldn't lug a physical book with them on an errand, but with e-books, it is possible to immerse oneself in Tolstoy while in line at the grocery store. Which choice best describes the relationship between the two paragraphs in the passage? A) The first paragraph introduces a topic about which the second paragraph goes into greater detail. B) The first paragraph offers a point of view that the second paragraph contradicts. C) The first paragraph describes a study while the second paragraph explains why that study was flawed. D) The first paragraph puts forward a point of view with which the second paragraph agrees, with some reservations. E) The first paragraph suggests that a topic requires further study while the second paragraph provides the desired information.
B) The first paragraph offers a point of view that the second paragraph contradicts. This is the best choice. The first paragraph suggests that people should read print books instead of e-books, while the second paragraph offers arguments in favor of e-books.
Shakespeare wrote four types of plays: histories, comedies, tragedies, and tragicomedies. Some scholars contend that Shakespeare's choice of three of these types of dramatic forms reflects his various psychological states. As a young man making a name for himself in London, he wrote comedies. Then, saddened by the death of his son, he turned to tragedies. Finally, seasoned by life's joys and sorrows, he produced tragicomedies. But a look at the theater scene of his day reveals that Shakespeare was not so much writing out of his heart as into his pocketbook. When comedies were the vogue, he wrote comedies; when tragedies were the rage, he wrote tragedies; and when tragicomedies dominated the stage, he produced tragicomedies. The primary purpose of the passage is to... A) examine Shakespeare's life in light of his dramatic works B) contest a theory that attempts to explain why Shakespeare wrote the kinds of plays he did C) explain the terms "comedy," "tragedy," and "tragicomedy" as they are used in discussions of Shakespeare's plays D) compare Shakespeare's plays with the works of other dramatists of his day E) discuss what is known about Shakespeare's psychological states
B) contest a theory that attempts to explain why Shakespeare wrote the kinds of plays he did This is the best choice. The first two sentences classify Shakespeare's plays into four categories and offer a theory, endorsed by "some scholars," concerning why Shakespeare chose to write three of these four kinds of plays. The next three sentences provide support for this theory by showing similarities between Shakespeare's likely psychological states and the plays he wrote at various times in his life. The word "But" in the next sentence indicates a change of direction in the passage: the author now suggests that the first theory may be wrong, and goes on to provide an alternate theory: that Shakespeare may well have written the kinds of plays he wrote not because they reflected a particular psychological state but because he thought they would be financially successful.
Predominantly Black land-grant colleges in the United States have a long tradition of supporting cooperative education programs. These programs combine academic courses with work experience that carries academic credit. This tradition has made these colleges the leaders in the recent movement in American education toward career-oriented curriculums. According to the passage, predominantly Black land-grant colleges in the United States are leaders in career-oriented education because they... A) have had cooperative education programs as part of their curriculums for many years B) were among the first colleges in the United States to shift away from career-oriented curriculums C) offer their students academic credit for their work experience prior to entering college D) have a long tradition of cooperation with local business and community leaders E) provide opportunities for students to work on campus to earn money for tuition
B) have had cooperative education programs as part of their curriculums for many years This choice best states the reason that the colleges are leaders in career-related education. The first sentence tells us that Black land-grant colleges have supported cooperative education programs "for a long time." The second sentence describes cooperative education programs. The final sentence tells us that it is this tradition of support for cooperative education programs that has made these colleges leaders in the career-oriented education movement.
In Understanding Media, Marshal McLuhan sheds a brilliant light, punctuated by occasional shadows of obscurity, on the essential nature of electronic media; the chapter on radio looks harder at that medium than anything since Arnheim's Radio. The phrase "shadows of obscurity" most probably refers to McLuhan's... A) use of imagery B) lack of clarity C) depth of understanding D) wide-ranging interests E) waning reputation
B) lack of clarity This is the best choice. The "brilliant light" shed by McLuhan is a figure of speech that can be interpreted as an eye-opening discussion of electronic media. The passage states that this brilliant light is "punctuated (interrupted) by" something else that contrasts with it. The "shadows of obscurity" can therefore be interpreted as confusing or unclear parts of McLuhan's discussion.
The product of an odd and an even number is even. The product of two even numbers is even. The product of "k" and "r" is odd. According to the statements above, which of the following is a valid conclusion?
Both "k" and "r" are odd. Having two even numbers or one odd and one even number would lead to an even product. Since we know the product is odd, both numbers must be odd.
Congruent triangles have...
Both the same shape and the same size; they have the same angle measures and the same side lengths (if two objects are congruent, then they are also similar)
The formula for "C," the circumference of the circle with radius "r," is...
C = 2πr
The formula for "C," the circumference of the circle with diameter "d," is...
C = πd
Which of the following represents four million two hundred forty thousand?
C) 4,240,000
Parents usually do not insist that their children learn to walk by a certain age. Parents feel confident that the children will learn to walk within a reasonable period of time, when their bodies are ready for such an undertaking. Teachers should adopt the same attitude when teaching children in school how to read. If teachers did this, children might learn to read much more quickly and experience less anxiety while doing so. The author's attitude toward teachers who try to force children to learn how to read once they reach a certain age can best be described as... A) sympathetic B) accepting C) disapproving D) neutral E) enthusiastic
C) disapproving This is the best choice. The word "should" in the third sentence indicates that the author is suggesting that, when teaching children how to read, teachers adopt the same attitude as that usually adopted by parents: not insisting that something be learned by a certain age. We can infer that the author would disapprove of teachers who try to force children to read at a certain age.
Archaeologists have found much evidence that many of the so-called "barbarians" of ancient times were actually part of socially complex cultures, and their primitive nature was often exaggerated by Roman writers. Essentially, they were the indigenous peoples who lived in lands beyond the direct control of Rome. During periods of expansion, barbarians of one decade might become provincial inhabitants of the Roman Empire of the ensuing decade. The last sentence of the passage serves to... A) differentiate the Roman perception of "barbarians" during periods of expansion versus periods of stagnation B) explain why certain Roman writers exaggerated the primitive nature of "barbarians" C) emphasize that the application of the term "barbarian" was fluid D) clarify the notion of "indigenous peoples" mentioned in the preceding sentence E) observe that archaeologists had exaggerated the complexity of the cultures of the "barbarians"
C) emphasize that the application of the term "barbarian" was fluid This is the best choice. The last sentence expresses that people who had been labeled barbaric "might become provincial inhabitants of the Roman Empire" when the borders changed, admitting new people within the boundaries of Rome.
The women's movement emerged in the United States in the 1830s, a period of intense reform and evangelism. Women were encouraged to speak out at religious revival meetings, and many women thus gained public speaking experience. When women sought and were denied leadership and the right to speak out in the abolitionist and temperance societies to which they belonged, they organized their own reform groups, and later worked to improve their own status. According to the passage, women formed their own reform societies because women... A) were denied membership in other reform societies B) disagreed with the aims of the societies to which they belonged C) were not permitted to act as leaders of the organizations of which they were members D) were preoccupied with issues that pertained only to the status of women E) wished to challenge the existing political order by questioning the political motives of their opponents
C) were not permitted to act as leaders of the organizations of which they were members This is the best choice. The last sentence states that women formed their own reform societies because they were "denied leadership and the right to speak out" in the societies to which they already belonged.
When asked to calculate "d" when "A" is given, it's best to...
Calculate "r" then multiply it by 2
Coordinating conjunctions
Can link independent clauses, FANBOYS is an acronym that can be useful to help you remember the seven coordinating conjunctions: for, and, nor, but, or, yet, so
Independent clauses
Can stand on their own as complete sentences
Subordinating conjunctions
Can turn independent clauses into dependent clauses, there are many subordinating conjunctions, common examples include: although, because, before, despite, while
Dependent clauses
Can't stand on their own as a complete sentences
Capitalization: tips and strategies
Capitalize by context: any word can require capitalization if it appears in the right context. Even if a word isn't usually capitalized, it needs to be capitalized if it appears in a proper noun phrase. Capitalize languages: the names of languages are both proper nouns and proper adjectives. They must be capitalized. Be careful with directions: take extra care with cardinal directions (north, southern). They are capitalized when they refer to a specific, named region (the American South). They aren't capitalized when they describe relative directions (the southern United States). Fix the common flaw: flawed capitalization is a common error type. If it is present, forget anything else that might seem weird to you about the sentence; the choice that fixes the common flaw will be the answer.
A recursive sequence generates each subsequent term by...
Carrying out the stated operations on the preceding term
Religious intolerance
Cemented by the Crusades and the Reconquista
Logical comparison
Comparisons must be made between two things of the same type
sugar
Columbus brought sugarcane to the Caribbean on his second voyage from Spain in 1493, and thereafter a wide variety of other herbs, flowers, seeds, and roots
Conventional expressions
Common phrases that must always be used in the same way or paired with the same words, they don't follow any one rule, but instead require familiarity with the patterns of Standard American English
What was one consequence of the implementation of Black Codes across the South immediately following the Civil War?
Congress passed the Fourteenth Amendment, which guaranteed African Americans citizenship and equal protection under the law. Since Black Codes attempted to restrict the rights of African Americans, in response, Congress passed the Fourteenth Amendment to ensure that their rights of citizenship were equally protected.
Shifts in verb tense: tips and strategies
Conjugate in context: to determine if an action happens in the past, present, or future, we'll need to rely upon context clues in the sentence. Common clues include dates, times, and other conjugated verbs. Place general facts in the present: the simple present tense is often used to describe general facts. If a statement is true regardless of timeframe, use the simple present tense. Fix the common flaw: flawed verb tense is a very common error type. If it is present, forget anything else that might seem weird to you about the sentence; the choice that fixes the common flaw will be the answer.
In which of the following are the numbers ordered from least to greatest?
D) -1, -1/2, 1/8, 1/3
Which of the following numbers has a 7 in the thousandths place?
D) 1,111.117
In order to decrease the value of a fraction, you can...
Decrease the numerator or increase the denominator
Verbs
Describe an action
Histograms
Divide the horizontal axis into equal-sized intervals and use the heights of the bars to show the count or percent of data within each interval; by convention, each interval includes the lower boundary but not the upper one, they show only totals for the intervals, not specific data points
Within-sentence punctuation: tips and strategies
Don't split subjects and verbs: unless two commas are being used to "fence off" a descriptive aside, no punctuation should come between a subject and verb. No commas before prepositions: unless a series of prepositions forms a list, commas shouldn't be placed in front of prepositional phrases. Lists of two don't need commas: a list of only two items doesn't require punctuation; commas are only needed when a list contains three or more items. Look for comma splices: a comma splice occurs when a comma links two independent clauses without a coordinating conjunction. Fix the common flaw: flawed punctuation is a very common error type; if it is present, forget anything else that might seem weird to you about the sentence; the choice that fixes the common flaw will be the answer.
How are quantitative data displayed?
Dotplot, histogram
William Bailey, an American Realist painter, studied at Yale in the 1950s. His still lifes depict smooth, rounded containers that sit in a field of uniform color. Bailey denies a close connection to Giorgio Morandi, an Italian painter of still lifes, but admits that they share "a belief in the power of the mute object." While Morandi painted from direct observation, Bailey painted from memory. This difference in method makes Bailey's objects superior to Morandi's, for they are thus purified, immutable, and mysterious. Which of the following statements, taken from the passage, is most clearly an expression of opinion rather than fact? A) William Bailey, an American Realist painter, studied at Yale in the 1950s. B) His still lifes depict smooth, rounded containers that sit in a field of uniform color. C) Bailey denies a close connection to Giorgio Morandi, an Italian painter of still lifes, but admits that they share "a belief in the power of the mute object." D) While Morandi painted from direct observation, Bailey painted from memory. E) This difference in method makes Bailey's objects superior to Morandi's, for they are thus purified, immutable, and mysterious.
E) This difference in method makes Bailey's objects superior to Morandi's, for they are thus purified, immutable, and mysterious. This is the best choice. There is no verifiable evidence that Bailey's artwork is objectively "superior to" Morandi's artwork. This is an expression of the author's personal preference, and as such is an opinion.
Sir Hans Sloane
Early botanists included this English naturalist who traveled to Jamaica in 1687 and there recorded hundreds of new plants
Lesson summary: The Spanish empire
Early interactions between the Spanish and Native Americans who lived in Central and South America led to a series of cultural exchanges that affected both the New World and the Old World.
Present participle
Ends in "-ing" and is used in progressive verbs, can serve as adjectives (for example: "falling snow") and can also serve as nouns (for example: "swimming is my favorite exercise")
The square of a number is...
Equal to the number multiplied by itself, for example: 3² = 3 x 3
pigs
Europeans introduced these, which they allowed to forage in forests and other wildlands; they consumed the foods on which deer and other indigenous species depended, resulting in scarcity of the game native peoples had traditionally hunted
Equivalent expressions
Expressions that work the same even though they look different, if two algebraic expressions are equivalent, then the two expressions have the same value when we plug in the same value(s) for the variable(s)
Pronoun agreement: tips and strategies
Find the antecedent: to recognize pronoun agreement errors, we must be able to identify the antecedent of the pronoun in question: the noun that the pronoun logically refers to. Isolate listed pronouns: when personal pronouns appear in a list, it becomes much harder to tell if they should be subject pronouns or object pronouns. If we get rid of the other list items and try the pronoun on its own, then errors in pronoun case will become clear. Fix the common flaw: flawed pronoun agreement is a very common error type. If it is present, forget anything else that might seem weird to you about the sentence; the choice that fixes the common flaw will be the answer.
Pronoun clarity: tips and strategies
Find the antecedent: to recognize pronoun clarity errors, we must be able to identify the antecedent of the pronouns in question. Ask yourself: what noun is this pronoun logically replacing? If you find multiple possible antecedents, or if you find no possible antecedents, then you've found a pronoun clarity error. Be specific: pronoun clarity errors often show up in questions that ask you to revise the underlined portion of a sentence. The easiest way to fix an error like this is usually to replace the unclear pronoun with the specific noun it was meant to represent. Fix the common flaw: flawed pronoun clarity is a very common error type. If it is present, forget anything else that might seem weird to you about the sentence; the choice that fixes the common flaw will be the answer.
Pentagon
Five-sided polygon
Frequently confused words: tips and strategies
Fix the common flaw: word confusion is a fairly common error type. If you see frequently confused words used incorrectly, forget anything else that might seem weird to you about the sentence; the choice that focuses on this flaw will be the answer.
Logical comparison: tips and strategies
Focus on what you can change: illogical comparisons often show up in questions that ask you to revise the underlined portion of a sentence. To find the most parallel choice, match what you can change to what you can't. Use your instincts: because comparison rules are based on conventional expressions and parallel phrasing, your instincts will serve you well. Does something feel unusual or awkward? If so, it's possible you've identified an error in logical comparison! Fix the common flaw: illogical comparison is a common error type. If it is present, forget anything else that might seem weird to you about the sentence; the choice that fixes the common flaw will be the answer.
Modifier placement: tips and strategies
Focus on what you can change: modifier placement errors often show up in questions that ask you to revise the underlined portion of a sentence. If you can't move the modifier, then move the noun. If you can't move the noun, then move the modifier. Fix the common flaw: flawed modifier placement is a very common error type. If it is present, forget anything else that might seem weird to you about the sentence; the choice that fixes the common flaw will be the answer.
Parallel structure: tips and strategies
Focus on what you can change: parallel structures often show up in questions that ask you to revise the underlined portion of a sentence. To find the most parallel choice, match what you can change to what you can't. Look for parallel structure "triggers:" long lists are easy to spot, but there are also a few phrases that require parallel structures: "both _ and _," "between _ and _," (n)either _ (n)or _," and "from _ to _." Fix the common flaw: parallel structure errors are very common on the Praxis Core Writing. If one is present, forget anything else that might seem weird about the sentence; the choice that fixes the common flaw will be the answer.
Verb tense
How a verb is conjugated, tells us when that action occurs (past, present, or future)
Apostrophe use
How an apostrophe ( ' ) can be used to show possession
Angle
Formed by two lines, line segments, or rays diverging from a vertex (common point) measured in degrees (°) which describe how spread apart intersecting lines or line segments are; narrow spreads have small angle measures, while wide spreads have large angle measures
Perpendicular
Forming a 90° angle
Subject-verb agreement: tips and strategies
Get rid of distracting phrases: the noun closest to the verb is not necessarily the subject! Subject-verb agreement errors can be hard to see when subjects and verbs don't appear directly next to one another. Try to simplify the sentence by eliminating any phrases that come between the subject and verb. This will make subject-verb agreement errors more obvious. Flip inverted sentences: sometimes, a subject comes after the verb. When this happens, subject-verb agreement errors can be hard to spot. If you notice an inverted sentence, try flipping the sentence so that the subject comes right before the verb. Compound subjects are plural subjects: it can be easy to miss subject-verb agreement errors when a verb has two singular subjects. Collective nouns are singular: it's easy to miss subject-verb agreement errors when collective nouns are paired with a plural verb. But remember: collective nouns (family, group, bunch) are singular. Fix the common flaw: flawed subject-verb agreement is a very common error type. If it is present, forget anything else that might seem weird to you about the sentence; the choice that fixes the common flaw will be the answer.
Clauses
Grammatical units that contain a subject and conjugated verb
Data representations
Graphics that display and summarize data and help us to understand the data's meaning
Redundant conjunctions
If both a subordinating and a coordinating conjunction are used to link the same clauses, an error is created
Examples of qualitative data
Hair color, languages spoken, favorite music genre
Equilateral triangles
Have three sides with the same length, each interior angle of an equilateral triangle measures 60°
Isosceles triangles
Have two sides with the same length, the two angles opposite these two sides have the same measure
Examples of quantitative data
Height, quiz score, distance traveled
Lesson summary: The Columbian Exchange
In 1492, Columbus landed in the Americas, starting a period of exploration and colonization
Treaty of Tordesillas
In 1494, Spain and Portugal moved the line of demarcation several degrees west, establishing Portugal's claim to Brazil
The associative law tells us that...
In addition or multiplication, we can associate the terms or factors as we please, for example: (a + b) + c = a + (b + c) (a x b) x c = a x (b x c)
smallpox
In the 1630s, half of the Huron and Iroquois people living near the Great Lakes died of this
Which of the following units of measure is most appropriate for stating the length of a pencil?
Inches
In order to increase the value of a fraction, you can...
Increase the numerator or decrease the denominator
Parts of a reference text
Index: gives page numbers by subject Glossary: offers definitions for key terms Abstract: provides a brief overview of a research project Bibliography/works cited: provide sources from which information was obtained Preface: introduces a subject and outlines goals for research
Probability
Indicates the chance that an event (a particular outcome of interest) will happen, can be any number from 0 to 1
Perfect squares
Integers whose square roots are also integers, for example: 9 is a perfect square because its square root, 3, is an integer; for numbers that aren't perfect squares, we write their square roots as radical expressions using the √ sign, for example: we would write the square root of 3 as √3
For numbers greater than or equal to 0, squaring and taking the square root are...
Inverse operations, this means if we perform both operations on a number in succession, the result would be the original number, for example: (√3)² = 3
Effect
Is usually a noun, "exhaustion has a negative effect on one's ability to drive"
Affect
Is usually a verb, "exhaustion negatively affects one's ability to drive"
1/2 xy = z In the formula above, if the value of x is doubled and the value of y is tripled, how will the value of z change?
It will be 6 times as great In order to preserve the equality, changes to the left side of the equation must be mirrored on the right side of the equation. The left side of the equation is 2 x 3 = 6 times as great, so the right side of the equation will also be 6 times as great.
Apostrophe use: tips and strategies
It's vs its: whenever you come across "it's" (with an apostrophe) get rid of the contraction and read it as "it is." When you get rid of contractions, usage errors become more obvious. Possessive pronouns: unlike possessive nouns, possessive pronouns never require an apostrophe. Possessive pronouns include yours, mine, its, theirs, hers, his, and whose. Fix the common flaw: flawed apostrophe use is a common error type. If it is present, forget anything else that might seem weird to you about the sentence; the choice that fixes the common flaw will be the answer.
Janice's family wants to have her grandmother's 80th birthday party at a restaurant. The restaurant charges $150 to rent a private room and $18 per person for food and beverages. If Janice's family has a budget of $700, how many people can they invite to the party?
Janice's family can invite 30 people to the party
A vertical bar chart
Lists the categories of the qualitative variable along a horizontal axis and uses the heights of the bars on the vertical axis to show the values of the quantitative variable
Semicolons ( ; ) should only be used to...
Link independent clauses without a conjunction
Subordination
Links two independent clauses by turning one into a dependent clause with the addition of a subordinating conjunction, requires a comma if the dependent clause comes first
Coordination
Links two independent clauses by using a comma and a coordinating conjunction
Factoring a number
Listing all its factors, the factors are usually presented in a list ordered from least to greatest
A horizontal bar chart
Lists the categories along the vertical axis and uses the lengths of the bars on the horizontal axis to show the values of the quantitative variable, this display draws attention to how the categories rank according to the amount of data within each
A pictograph
Like a horizontal bar chart, but uses pictures instead of the lengths of bars to represent the values of the quantitative variable, each picture represents a certain quantity, and each category can have multiple pictures; are visually interesting, but require us to use the legend to convert the number of pictures to quantitative values.
How are trends over time displayed?
Line graphs (a set of points connected by line segments) are data displays that show trends over time, these graphs typically present time (day, month, or year) on the horizontal axis and another quantitative variable (temperature, oil price, or income) on the vertical axis; each dot on a line graph represents the value of a quantitative variable at a particular time, and the dots are connected to form graph
Adjective/adverb confusion: tips and strategies
Look for "-ly" endings: if you're struggling to identify the difference between a word's adjective and adverb forms, just remember: most adverbs end in "-ly." For instance, "easy" is an adjective, while "easily" is the equivalent adverb. Fix the common flaw: adjective/adverb confusion is a common error type. If it is present, forget anything else that might seem weird to you about the sentence; the choice that fixes the common flaw will be the answer.
Negation: tips and strategies
Look for split negatives: double negatives can be harder to see if they're split up by a verb phrase or a descriptive aside. Any time a negative word is underlined, comb closely through the entire sentence to see if another negative word or a "barely" word is hiding anywhere. Fix the common flaw: negation is a common error type. If a negation error is present, forget anything else that might seem weird to you about the sentence; the choice that fixes the common flaw will be the answer.
Concision: tips and strategies
Look out for frequency words: one easily-overlooked redundancy concerns the use of adverbs that describe how frequently an action occurs. When you see words like "annual," "monthly," or "often," make sure there aren't any other adverbs that also describe how frequently the action occurs. Fix the common flaw: concision is a common error type. If a redundancy is present, forget anything else that might seem weird to you about the sentence; the choice that fixes the common flaw will be the answer.
Standard deviation
Measures the typical spread from the mean, the average distance between the mean and a data point, larger values of standard deviation indicate greater spread in the data
Straight angles
Measure 180°
Right angles
Measure 90°
Obtuse angles
Measure greater than 90° and less than 180°
Acute angles
Measure less than 90°
Range
Measures the total spread of the data, the difference between the highest and lowest values, range = highest value - lowest value, a larger range indicates a greater spread in the data
Silk Road
Merchants' ships brought Europeans valuable goods, traveling between the port cities of western Europe and the East from the 10th century on along these collective routes, transporting goods along it was costly, slow, and unprofitable
xocolatl
Mesoamerican Indians consumed unsweetened chocolate in a drink with chili peppers, vanilla, and a spice called achiote; it was part of ritual ceremonies like marriage
Relative clauses
Modifiers beginning with "who," "when," "which," "where," or "that;" they must immediately follow the noun they describe
Dividing by a fraction is equivalent to...
Multiplying by the reciprocal of that fraction
Any changes to one side of the equation...
Must also be applied to the other side of the equation
Introductory modifying phrases
Must be immediately followed by the noun they logically describe
Comma-bound descriptive asides
Must be placed next to the noun they logically describe
Mode
Number that appears the most often in the set
Double negatives
Occur when multiple negative words are used redundantly
Negation
Occurs when the meaning of a statement is altered by the addition of negative words like not, never, and no
syphilis
One disease did travel the other direction, this lethal sexually transmitted disease, came with travelers from the New World to Europe for the first time
Inverse operations
Operations that "undo" each other: addition and subtraction are inverse operations, multiplication and division are inverse operations
Which of the following statements must be true for a square?
Opposite sides have equal length and Adjacent angles have the same measure and Adjacent sides are perpendicular
Perimeter of a rectangle
P = 2(l + w)
Perimeter of a square
P = 4s
The probability that event A will happen is written as...
P(A)
Frequently confused words
Pairs of words that either look and sound similar ("than" and "then") or have similar but distinct meanings ("fewer" and "less")
Rectangles
Parallelograms with four 90° angles, the adjacent sides are perpendicular; while all rectangles are parallelograms, not all parallelograms are rectangles
Squares
Parallelograms with four sides of equal length and four 90° angles; while all squares are both rectangles and parallelograms, not all parallelograms are squares and not all rectangles are squares
Percent (%)
Parts per hundred (100)
Complementary percentages
Percentages that add up to 100%, for example: 60% + 40% = 100%
To reverse a solution procedure, we...
Perform the inverse operations in reverse order
Quadrilaterals
Polygons with four sides and four interior angles
Triangles
Polygons with three sides and three interior angles
Elmina Castle
Portuguese fort located in modern-day Ghana; became more of a holding pen for enslaved Africans from the interior of the continent than a trading post, as the markets for slave labor in both Europe and then the New World boomed
Parallelograms
Quadrilaterals with two pairs of parallel sides and two pairs of angles with the same measure, the opposite sides have the same length, and adjacent angles are supplementary
Pre-algebra word problems
Questions that require translating sentences to expressions, then evaluating those expressions
Conversion factors
Ratios of two equivalent measurements in different units
Equivalent ratios
Ratios that have the same value
Source-based essay: tips and strategies
Read closely: success on the source-based essay relies on how well you engage with the provided sources. Don't be in a rush to start writing! Take a few minutes to read the sources closely so that they are reflected accurately in your essay. Compare the sources: the sources will provide two separate perspectives on the same issue. On what do the sources agree? On what do they disagree? By thinking and writing about the sources in this way, you will greatly increase your ability to identify the most important aspects of the issue under discussion. Introduce original analysis: success on the source-based essay is primarily based on how well you use your sources, but don't just include summaries and quotations. Use your own words and ideas to supplement and evaluate the information included in the sources. Doing so will improve your essay and increase your score. Proofread: even if you're not quite done writing, use the last minute or two of your allotted time to proofread your essay. While your essay isn't expected to be highly polished, fixing even a few grammar errors can make a sizable impact on your essay score.
Perfect tense verbs
Refer to a completed action
Progressive tense verbs
Refer to an ongoing action
Modifier placement
Refers to the fact that modifiers must appear directly next to the nouns they logically describe
A shortcut for converting percentages to decimals is to...
Remove the % symbol and move the decimal point 2 places to the left, for example: to convert 25% to a decimal, 1) remove the % symbol → 25, 2) move the decimal point left 2 places → 0.25
Diameter
Represented by "d," is twice as long as the radius, d = 2r
Subject-verb agreement
Requires that a subject and verb agree in number
How are relationships between variables displayed?
Scatterplots (a graph where values of two variables are plotted along two axes) display data about two quantitative variables as a set of points in the xy-plane; this is a key tool to determine if there is a relationship between the values of two variables
Commas ( , ) should only be used to...
Separate list items, separate descriptive asides from the sentence, link dependent clauses to independent clauses, link independent clauses with help from a coordinating conjunction
Counterexamples
Showing that a mathematical statement is true requires a formal proof; however, showing that a mathematical statement is false only requires us to find one example where the statement isn't true, such an example is called a counterexample because it is an example that counters, or goes against, the statement's conclusion
"mourning wars"
Some indigenous people perceived disease as a weapon used by hostile spiritual forces, and they went to war to exorcise the disease from their midst; these "wars" in eastern North America were designed to gain captives who would either be adopted or ritually tortured and executed to assuage the anger and grief caused by loss
Seven Years War (1754-1763)
Sometimes called the French and Indian War, it was a conflict between France and Britain, in which the Algonquins sided with the French and the Iroquois sided with the British and the colonists
Islam
Spread throughout the Middle East and into Europe until 732
If a square is divided into 4 identical areas, which of the following could be the shapes of these areas?
Squares and Rectangles and Triangles
The triangle inequality rule
States that the longest side of a triangle must be shorter than the combined lengths of the two other sides; in other words, for a triangle with side lengths a, b, and c: a + b > c
A table
Summarizes the data using rows and columns, each column contains data for a single variable, and a basic table contains one column for the qualitative variable and one for the quantitative variable, each row contains a category of the qualitative variable and the corresponding value of the quantitative variable
How are qualitative data displayed?
Table, vertical bar chart, pictograph, circle graph (pie chart)
The Supreme Court case Dred Scott v. Sanford, which declared that black Americans were not US citizens, was ultimately overruled by...
The Fourteenth Amendment. The Fourteenth Amendment enshrined the citizenship status of black Americans into the Constitution.
Research skills
The abilities required to successfully interact with scholarly articles, reference texts, and citations
Vertical angles
The angles on the opposite sides of an intersection of two lines, they have the same measure
The distributive law deals with...
The combination of addition and multiplication, when a sum is multiplied by a value, the value is distributed to each part of the sum, for example: a (b + c) = a x b + a x c
Concision
The concept of eliminating unnecessary words, the purpose of concision is to avoid redundancy, if two words mean the same thing, we shouldn't use both of them
Pronoun clarity
The idea that a pronoun must clearly refer to a logical antecedent
Pronoun agreement
The idea that pronouns should be similar to the nouns they represent
Parallel structure
The idea that when two or more words, phrases, or clauses are linked, they should share the same structure
The greatest common factor of two numbers is...
The largest whole number that both numbers are divisible by
Antecedent
The noun that a pronoun replaces
overhunting
The popularity of beaver-trimmed hats in Europe, coupled with Native Americans' desire for European weapons, led to this of beavers in the Northeast
Columbian Exchange
The process by which commodities (horses, tomatoes, sugar), people, and diseases crossed the Atlantic
Subordination and coordination
The process of linking clauses with punctuation and conjunctions
Noun agreement
The requirement that a noun agree in number with the logic of the sentence
Similar triangles have...
The same shape, but not necessarily the same size; they have the same angle measures, but not the same side lengths
Within-sentence punctuation
The separation of sentence elements using commas and semicolons; used to format lists, link clauses, and clarify sentence structure
Hypotenuses
The sides opposite the right angles in right triangles
The least common multiple of two numbers is...
The smallest whole number that is divisible by both numbers
Epidemic
The spread of a disease to a large group of people within a population in a short period of time
sugar production
The travels of Portuguese traders to western Africa also acquainted the Portuguese with the African slave trade, already widely in practice in West Africa and funded by this on the newly colonized Atlantic islands; upon discovering the immense global market for sugar, the Portuguese began to trade enslaved people across the Atlantic to toil on the sugar plantations
Spread describes...
The variation of the data; two measures of spread are range and standard deviation.
Argumentative essay: tips and strategies
Thesis first: after reading the prompt, start by drafting a clear thesis statement. If you start with your thesis, your essay will be clear and focused, and you'll be less likely to introduce contradictory arguments. Plan ahead: once you've decided on a thesis, take a minute to choose 3 - 4 examples that directly support your argument. If you start writing before outlining your support, the resulting essay may be confusing or disorganized. Make sure you know your destination before you start the journey! Be specific: essays with highly specific examples receive much better scores than those with broad or vague examples. Even if you're introducing the exact same support, "some people often..." (vague) won't score as highly as "once, my sister..." (specific). Proofread: even if you're not quite done writing, use the last minute or two of your allotted time to proofread your essay. While your essay isn't expected to be highly polished, fixing even a few grammar errors can make a sizable impact on your essay score.
Argumentative essay
This essay task will prompt you to discuss the extent to which you agree or disagree with a commonly held opinion. The topic is designed to be familiar to all test-takers, so responding to it won't require specialized knowledge. You should approach the prompt by drawing on your personal experiences, observations, or past reading to support your position with specific reasons and examples. Your essay will be evaluated by two graders on a scale of 1 to 6. Those scores will be averaged and incorporated into your overall Writing score. A 6 essay demonstrates a high degree of competence in response to the assignment but may have a few minor errors. An essay in this category: states or clearly implies the writer's position or thesis; organizes and develops ideas logically, making insightful connections between them; clearly explains key ideas, supporting them with well-chosen reasons, examples, or details; displays effective sentence variety; clearly displays facility in the use of language; is generally free from errors in grammar, usage, and mechanics.
Source-based essay
This essay task will prompt you to extract information from two provided sources to identify important concerns related to an issue. While the topic should be somewhat familiar, the sources will be highly specific and academic. You will need to read the sources closely to gain insight into the topic. You should approach the prompt by drawing heavily from both sources and engaging directly with the arguments they contain. Your essay will be evaluated by two graders on a scale of 1 to 6. Those scores will be averaged and incorporated into your overall Writing score. A 6 essay demonstrates a high degree of competence in response to the assignment but may have a few minor errors. An essay in this category: insightfully explains why the concerns are important, supporting the explanation with effective links between the two sources and well-chosen reasons, examples, or details; incorporates information from both sources to identify and explain important concerns regarding the issue discussed in the sources; organizes and develops ideas logically; displays effective sentence variety; clearly displays facility in the use of language; is generally free from errors in grammar, usage, and mechanics; cites both sources when paraphrasing or quoting.
microbes
Travelers between the Americas, Africa, and Europe also included these: silent, invisible life forms that had profoundly devastating consequences; native inhabitants had no immunity, they caused sickness and death everywhere Europeans settled
Treaty of Paris (1763)
Treaty ending the Seven Years War, in which French ceded much of its North American territory to the British
Research skills: tips and strategies
Trust yourself: rules tested in research skills questions can be complex, but you don't need to know the rules in-depth to have success. Rely on your past experience doing research and citing sources. The instincts you've already developed will serve you well. Don't sweat it: research skills questions cover a wide variety of topics, knowledge, and skills. But don't spend much time learning all the details. There are only four research skills questions on each Praxis Core Writing test, so they make up a very small portion of the exam.
What consequences did members of the 7th US Cavalry undergo for participating in the massacre at Wounded Knee?
Twenty US Army personnel were awarded the Medal of Honor, and the event was generally regarded as favorable by the American public. The American public celebrated the massacre of hundreds of Sioux men, women, and children, as the United States marked the definitive end of Indian resistance to white settlement.
A transversal of two parallel lines creates...
Two sets of angles with identical angle measures at the intersections
What did the Germans use to sink the Lusitania (and other ships during World War I)?
U-boats U-boats, or German submarines, became critical for combat during World War I.
Dotplots
Use one dot for each data point, the dots are plotted above their corresponding values on a number line, the number of dots above each specific value represents the count of that value; they show the value of each data point and are practical for small data sets
Subordination and coordination: tips and strategies
Use the logical conjunction: conjunctions express the relationship between the clauses being linked. Make sure all conjunctions logically reflect this relationship. Transitions aren't conjunctions: look out for transition words that don't actually serve as conjunctions. While "therefore" and "so" might mean the same thing, only "so" can be used as a conjunction. Fix the common flaw: flawed subordination and coordination is a common error type. If it is present, forget anything else that might seem weird to you about the sentence; the choice that fixes the common flaw will be the answer.
Noun agreement: tips and strategies
Use the same number for descriptions: verbs are often used to describe their subjects. In these instances, the description must agree with the subject in number. Other times, descriptive asides will appear next to nouns. Those descriptions must also agree in number with the nouns they modify. Look for compound noun phrases: when two singular nouns come together, they create a plural noun phrase that only agrees with plural nouns. Fix the common flaw: flawed noun agreement is a common error type. If it is present, forget anything else that might seem weird to you about the sentence; the choice that fixes the common flaw will be the answer.
Conventional expressions: tips and strategies
Use your instincts: unfortunately, there is no rule or formula that can be applied to conventional expressions. However, because conventions are based on how words and phrases are commonly used, your instincts will serve you well. Does something feel unusual or awkward? If so, it's possible you've identified an error in convention! Fix the common flaw: flawed conventional expressions is a very common error type. If it is present, forget anything else that might seem weird to you about the sentence; the choice that fixes the common flaw will be the answer.
Adjectives
Used to describe nouns
Adverbs
Used to describe verbs and adjectives
Median
Useful to describe the center of data with outliers, the middle value when the data are ordered from least to greatest, if the number of values is odd, the median is the middle value, if the number of values is even, the median is the average of the two middle values
Volume of a rectangular solid
V = lwh
Outliers
Values that are much higher or lower than the others
Participles
Verb forms that can't serve as the main verb of a clause without a helping verb
A solution procedure is a...
Verbal description of the steps we take to solve an equation
Maritime trade
Was dramatically increased by the Crusades between the East and West
The commutative law tells us...
We can reorder the terms when performing addition or multiplication, for example: a + b = b + a a x b = b x a
The square root of a positive number x, written as √x, is the answer to the question...
What positive number squared equals x? For example: since 3² = 9, the square root of 9, or √9, is 3
Prime numbers
Whole numbers greater than 1 whose only factors are 1 and itself, for example: 17 is prime because it is only divisible by 1 and 17
Factors
Whole numbers that divide evenly (no remainder) into another whole number
Modifiers
Words or phrases that describe a noun
Prepositions
Words that express a relationship in time or space to the noun they are followed by, common prepositions include: of, for, with, about, from, to, at, in, on, before, between, above, around, and many more. Often, a given word must be paired with a specific preposition or an error is created. Look out for: "(n)either _ (n)or _"
Conjunctions
Words that express the relationship between clauses
Prime factorization
Writing a number as a product of factors that are all prime numbers, for example: the prime factorization of 30 is 2 x 3 x 5, it is helpful when finding the greatest common factor or least common multiple, but is not the only way to do it
cash crop
a crop cultivated for sale instead of personal consumption
Colonial mercantilism
a set of protectionist policies designed to benefit the colonizing nation, relied on several factors: Colonies rich in raw materials, Cheap labor, Colonial loyalty to the home government, Control of the shipping trade
The Columbian Exchange embodies...
both the positive and negative environmental and health results of contact as well as the cultural shifts produced by such contact
Muslim middlemen
collected taxes as the goods changed hands along the silk road
The circumference of a circle is the...
distance around it, it's usually represented by "C," we can calculate "C" using either "d" or "r"
Colonizers means of demarcating private property included...
erecting fields and fences
Tobacco
it became the first truly global commodity; English, French, Dutch, Spanish, and Portuguese colonists all grew it for the world market, became a valuable export as the habit of smoking took hold in Europe
The triangle trade
merchants on each continent sent commodities and people between Europe, Africa, and the Americas
p% is equivalent to...
p:100 and p/100
Commodification
quickly affected production in the New World: American silver, tobacco, and other items (which were used by native peoples for ritual purposes) became European commodities with monetary value, This process disrupted native economies and spurred early commercial capitalism
Circles are...
round, they don't have any corners, all of the points on a circle are the same distance from the center (that distance is called the radius, it's usually represented by "r")
Prince Henry the Navigator
spearheaded Portugal's exploration of Africa and the Atlantic in the 1400s
Christopher Columbus
started a colonization project for The Spanish who were threatened by the Portuguese monopoly on enslaved Africans and expansion in the Atlantic in 1492
The task of cataloging the new plants found in America led to...
the emergence of the science of botany
If p = mv, which of the following correctly shows v in terms of p and m?
v = p/m
P(A) =
ways A can happen/possible outcomes
Which of the following would be a logical and correct first step in solving the equation 4x + 8 = 20?
x + 2 = 5 This choice divides both sides of the equation by 4
Which of the following statements are true for all real numbers x, y, and z?
x + y + z = y + (x + z) x (y - z) = xy - xz
2 (3x − 1) = 7x − 3 What is the solution to the equation above?
x = 1
The area of a circle is 100 square centimeters. What is the radius of the circle in centimeters?
√100/π
For triangles LMN and PQR, LM = PQ and ∠M has the same measure as ∠Q. Which of the following statements, if true, is sufficient to show that the two triangles are congruent?
∠L and ∠P have the same measure. This choice, along with the given information, gives us enough information to meet the ASA congruence criterion. ∠N and ∠R have the same measure. This choice, along with the given information, gives us enough information to meet the AAS congruence criterion. MN = QR This choice, along with the given information, gives us enough information to meet the SAS congruence criterion.